Pharmacology > EXAM REVIEW > katzung board review with complete solution latest 2020 (All)

katzung board review with complete solution latest 2020

Document Content and Description Below

katzung board review INTRODUCTION 1. A 3-year-old is brought to the emergency department having just ingested a large overdose of tolbutamide... , an oral antidiabetic drug. Tolbutamide is a weak acid with a pKa of 5.3. It is capable of entering most tissues, including the brain. On physical examination, the heart rate is 100/min, blood pressure 90/50 mm Hg, and respiratory rate 20/min. Which of the following statements about this case of tolbutamide overdose is most correct? (A) Urinary excretion would be accelerated by administration of NH4Cl, an acidifying agent (B) Urinary excretion would be accelerated by giving NaHCO3, an alkalinizing agent (C) Less of the drug would be ionized at blood pH than at stomach pH (D) Absorption of the drug would be slower from the stomach than from the small intestine (E) Hemodialysis is the only effective therapy 2. Botulinum toxin is a large protein molecule. Its action on cholinergic transmission depends on an intracellular action within nerve endings. Which one of the following processes is best suited for permeation of very large protein molecules into cells? (A) Aqueous diffusion (B) Endocytosis (C) First-pass effect (D) Lipid diffusion (E) Special carrier transport 3. A 12-year-old child has bacterial pharyngitis and is to receive an oral antibiotic. She complains of a sore throat and pain on swallowing. The tympanic membranes are slightly reddened bilaterally, but she does not complain of earache. Blood pressure is 105/70 mm Hg, heart rate 100/mm, temperature 37.8 C (100.1 F). Ampicillin is a weak organic acid with a pKa of 2.5. What percentage of a given dose will be in the lipid-soluble form in the duodenum at a pH of 4.5? (A) About 1% (B) About 10% (C) About 50% (D) About 90% (E) About 99% 4. Ampicillin is eliminated by first-order kinetics. Which of the following statements best describes the process by which the plasma concentration of this drug declines? (A) There is only 1 metabolic path for drug elimination (B) The half-life is the same regardless of the plasma concentration (C) The drug is largely metabolized in the liver after oral administration and has low bioavailability (D) The rate of elimination is proportional to the rate of administration at all times (E) The drug is distributed to only 1 compartment outside the vascular system 5. The pharmacokinetics of a new drug are under study in a phase 1 clinical trial. Which statement about the distribution of drugs to specific tissues is most correct? (A) Distribution to an organ is independent of blood flow (B) Distribution is independent of the solubility of the drug in that tissue (C) Distribution into a tissue depends on the unbound drug concentration gradient between blood and the tissue (D) Distribution is increased for drugs that are strongly bound to plasma proteins (E) Distribution has no effect on the half-life of the drug 6. The pharmacokinetic process or property that distinguishes the elimination of ethanol and high doses of phenytoin and aspirin from the elimination of most other drugs is called (A) Distribution (B) Excretion (C) First-pass effect (D) First-order elimination (E) Zero-order elimination 7. A new drug was administered intravenously, and its plasma levels were measured for several hours. A graph was prepared as shown below, with the plasma levels plotted on a logarithmic ordinate and time on a linear abscissa. It was concluded that the drug has first-order kinetics. From this graph, what is the best estimate of the half-life? (A) 0.5 h (B) 1 h (C) 3 h (D) 4 h (E) 7 h 8. A large pharmaceutical company has conducted extensive animal testing of a new drug for the treatment of advanced prostate cancer. The chief of research and development recommends that the company now submit an IND application in order to start clinical trials. Which of the following statements is most correct regarding clinical trials of new drugs? (A) Phase 1 involves the study of a small number of normal volunteers by highly trained clinical pharmacologists (B) Phase 2 involves the use of the new drug in a large number of patients (1000–5000) who have the disease to be treated under conditions of proposed use (eg, outpatients) (C) Chronic animal toxicity studies must be complete and reported in the IND (D) Phase 4 involves the detailed study of toxic effects that have been discovered in phase 3 (E) Phase 2 requires the use of a positive control (a known effective drug) and a placebo 9. Which of the following statements about animal testing of potential new therapeutic agents is most correct? (A) Extends at least 3 years to discover late toxicities (B) Requires at least 1 primate species (eg, rhesus monkey) (C) Requires the submission of histopathologic slides and specimens to the FDA for evaluation by government scientists (D) Has good predictability for drug allergy-type reactions (E) May be abbreviated in the case of some very toxic agent 10. The “dominant lethal” test involves the treatment of a male adult animal with a chemical before mating; the pregnant female is later examined for fetal death and abnormalities. The dominant lethal test therefore is a test of (A) Teratogenicity (B) Mutagenicity (C) Carcinogenicity (D) Sperm viability 11. Which of the following would probably not be included in an optimal phase 3 clinical trial of a new analgesic drug for mild pain? (A) A negative control (placebo) (B) A positive control (current standard analgesic therapy) (C) Double-blind protocol (in which neither the patient nor immediate observers of the patient know which agent is active) (D) A group of 1000–5000 subjects with a clinical condition requiring analgesia (E) Prior submission of an NDA (new drug application) to the FDA 12. Which of the following statements about the testing of new compounds for potential therapeutic use in the treatment of hypertension is most correct? (A) Animal tests cannot be used to predict the types of clinical toxicities that may occur because there is no correlation with human toxicity (B) Human studies in normal individuals will be done before the drug is used in individuals with hypertension (C) The degree of risk must be assessed in at least 3 species of animals, including 1 primate species (D) The animal therapeutic index must be known before trial of the agents in humans 13. The Ames test is frequently carried out before clinical trials are begun. The Ames test is a method that detects (A) Carcinogenesis in primates (B) Carcinogenesis in rodents (C) Mutagenesis in bacteria (D) Teratogenesis in any mammalian species (E) Teratogenesis in primates 14. Which of the following statements about new drug development is most correct? (A) Drugs that test positive for teratogenicity, mutagenicity, or carcinogenicity can be tested in humans (B) Food supplements and herbal (botanical) remedies are subject to the same FDA regulation as ordinary drugs (C) All new drugs must be studied in at least 1 primate species before NDA submission (D) Orphan drugs are drugs that are no longer produced by the original manufacturer (E) Phase 4 (surveillance) is the most rigidly regulated phase of clinical drug trials PHARMACODYNAMICS 1. A 55-year-old woman with hypertension is to be treated with a thiazide diuretic. Thiazide A in a dose of 5 mg produces the same decrease in blood pressure as 500 mg of thiazide B. Which of the following statements best describes these results? (A) Thiazide A is more efficacious than thiazide B (B) Thiazide A is about 100 times more potent than thiazide B (C) Toxicity of thiazide A is less than that of thiazide B (D) Thiazide A has a wider therapeutic window than thiazide B (E) Thiazide A has a longer half-life than thiazide B 2. Graded and quantal dose-response curves are being used for evaluation of a new antiasthmatic drug in the animal laboratory and in clinical trials. Which of the following statements best describes graded dose-response curves? (A) More precisely quantitated than quantal dose-response curves (B) Obtainable from isolated tissue preparations but not from the study of intact subjects (C) Used to determine the maximal efficacy of the drug (D) Used to determine the therapeutic index of the drug (E) Used to determine the variation in sensitivity of subjects to the drug 3. Prior to clinical trials in patients with heart failure, an animal study was carried out to compare two new positive inotropic drugs (A and B) to a current standard agent (C). The results of cardiac output measurements are shown in the graph below. Which of the following statements is correct? (A) Drug A is most effective (B) Drug B is least potent (C) Drug C is most potent (D) Drug B is more potent than drug C and more effective than drug A (E) Drug A is more potent than drug B and more effective than drug C 4. A study was carried out in isolated intestinal smooth muscle preparations to determine the action of a new drug “novamine,” which in separate studies bound to the same receptors as acetylcholine. In the absence of other drugs, acetylcholine caused contraction of the muscle. Novamine alone caused relaxation of the preparation. In the presence of a low concentration of novamine, the EC50 of acetylcholine was unchanged, but the Emax was reduced. In the presence of a high concentration of novamine, extremely high concentrations of acetylcholine had no effect. Which of the following expressions best describes novamine? (A) A chemical antagonist (B) An irreversible antagonist (C) A partial agonist (D) A physiologic antagonist (E) A spare receptor agonist 5. Beta adrenoceptors in the heart regulate cardiac rate and contractile strength. Several studies have indicated that in humans and experimental animals, about 90% of β adrenoceptors in the heart are spare receptors. Which of the following statements about spare receptors is most correct? (A) Spare receptors, in the absence of drug, are sequestered in the cytoplasm (B) Spare receptors may be detected by finding that the drug-receptor interaction lasts longer than the intracellular effect (C) Spare receptors influence the maximal efficacy of the drug-receptor system (D) Spare receptors activate the effector machinery of the cell without the need for a drug (E) Spare receptors may be detected by the finding that the EC50 is smaller than the Kd for the agonist 6. Two cholesterol-lowering drugs, X and Y, were studied in a large group of patients, and the percentages of the group showing a specific therapeutic effect (35% reduction in lowdensity lipoprotein [LDL] cholesterol) were determined. The results are shown in the following table. Which of the following statements about these results is correct? (A) Drug X is safer than drug Y (B) Drug Y is more effective than drug X (C) The 2 drugs act on the same receptors (D) Drug X is less potent than drug Y (E) The therapeutic index of drug Y is 10 7. Sugammadex is a new drug that reverses the action of rocuronium and certain other skeletal muscle-relaxing agents (nondepolarizing neuromuscular blocking agents). It appears to interact directly with the rocuronium molecule and not at all with the rocuronium receptor. Which of the following terms best describes sugammadex? (A) Chemical antagonist (B) Noncompetitive antagonist (C) Partial agonist (D) Pharmacologic antagonist (E) Physiologic antagonist DIRECTIONS: 8–10. Each of the curves in the graph below may be considered a concentration-effect curve or a concentration-binding curve. 8. Which of the curves in the graph describes the percentage of binding of a large dose of full agonist to its receptors as the concentration of a partial agonist is increased from low to very high levels? (A) Curve 1 (B) Curve 2 (C) Curve 3 (D) Curve 4 (E) Curve 5 9. Which of the curves in the graph describes the percentage effect observed when a large dose of full agonist is present throughout the experiment and the concentration of a partial agonist is increased from low to very high levels? (A) Curve 1 (B) Curve 2 (C) Curve 3 (D) Curve 4 (E) Curve 5 10. Which of the curves in the graph describes the percentage of binding of the partial agonist whose effect is shown by Curve 4 if the system has many spare receptors? (A) Curve 1 (B) Curve 2 (C) Curve 3 (D) Curve 4 (E) Curve 5 PHARMACOKINETICS 1. Mr Jones has zero kidney function and is undergoing hemodialysis while awaiting a kidney transplant. He takes metformin for type 2 diabetes mellitus and was previously stabilized (while his kidney function was adequate) at a dosage of 500 mg twice daily, given orally. The plasma concentration at this dosage with normal kidney function was found to be 1.4 mg/L. He has been on dialysis for 10 days and metformin toxicity is suspected. A blood sample now shows a metformin concentration of 4.2 mg/L. What was Mr. Jones’ clearance of metformin while his kidney function was normal? (A) 238 L/d (B) 29.8 L/h (C) 3 L/d (D) 238 L/h (E) 30 L/min 2. Ms Smith, a 65-year-old woman with pneumonia, was given tobramycin, 150 mg, intravenously. After 20 minutes, the plasma concentration was measured and was found to be 3 mg/L. Assuming no elimination of the drug in 20 minutes, what is the apparent volume of distribution of tobramycin in Ms Smith? (A) 3 L/min (B) 3 L (C) 50 L (D) 7 L (E) 0.1 mg/min 3. St John’s Wort, a popular botanical remedy, is a potent inducer of hepatic phase I CYP3A4 enzymes. Verapamil and phenytoin are both eliminated from the body by metabolism in the liver. Verapamil has a clearance of 1.5 L/min, approximately equal to liver blood flow, whereas phenytoin has a clearance of 0.1 L/min. Based on this fact, which of the following is most correct? (A) St John’s Wort will increase the half-life of phenytoin and verapamil (B) St John’s Wort will decrease the volume of distribution of CYP3A4 substrates (C) St John’s Wort will decrease the hepatic extraction of phenytoin (D) St John’s Wort will decrease the first-pass effect for verapamil (E) St John’s Wort will increase the clearance of phenytoin 4. A 55-year-old man with severe rheumatoid arthritis has elected to participate in the trial of a new immunosuppressive agent. It is given by constant intravenous infusion of 8 mg/h. Plasma concentrations (Cp) are measured with the results shown in the following table. What conclusion can be drawn from these data? (A) Clearance is 2 L/h (B) Doubling the rate of infusion would result in a plasma concentration of 16 mg/L at 40 h (C) Elimination follows zero-order kinetics (D) Half-life is 8 h (E) Volume of distribution is 30 L 5. You are the only physician in a clinic that is cut off from the outside world by violent storms, flooding, and landslides. A 15-year-old girl is brought to the clinic with severe asthmatic wheezing. Because of the lack of other drugs, you decide to use intravenous theophylline for treatment. The pharmacokinetics of theophylline include the following average parameters: Vd 35 L; CL 48 mL/min; half-life 8 h. If an intravenous infusion of theophylline is started at a rate of 0.48 mg/min, how long would it take to reach 93.75% of the final steadystate concentration? (A) Approximately 48 min (B) Approximately 7.4 h (C) Approximately 8 h (D) Approximately 24 h (E) Approximately 32 h 6. A 74-year-old retired mechanic is admitted with a myocardial infarction and a severe acute cardiac arrhythmia. You decide to give lidocaine to correct the arrhythmia. A continuous intravenous infusion of lidocaine, 1.92 mg/min, is started at 8 am. The average pharmacokinetic parameters of lidocaine are: Vd 77 L; clearance 640 mL/min; half-life 1.4 h. What is the expected steady-state plasma concentration? (A) 40 mg/L (B) 3.0 mg/L (C) 0.025 mg/L (D) 7.2 mg/L (E) 3.46 mg/L 7. A new drug is under study in phase 1 trials. It is found that this molecule is avidly taken up by extravascular tissues so that the final total amount in the extravascular compartment at steady state is 100 times the amount remaining in the blood plasma. What is the probable volume of distribution in a hypothetical person with 8 L of blood and 4 L of plasma? (A) Insufficient data to calculate (B) 8 L (C) 14.14 L (D) 100 L (E) 404 L 8. A 63-year-old woman in the intensive care unit requires an infusion of procainamide. Its half-life is 2 h. The infusion is begun at 9 am. At 1 pm on the same day, a blood sample is taken; the drug concentration is found to be 3 mg/L. What is the probable steady-state drug concentration after 16 or more hours of infusion? (A) 3 mg/L (B) 4 mg/L (C) 6 mg/L (D) 9.9 mg/L (E) 15 mg/L 9. A 30-year-old man is brought to the emergency department in a deep coma. Respiration is severely depressed and he has pinpoint pupils. His friends state that he self-administered a large dose of morphine 6 h earlier. An immediate blood analysis shows a morphine blood level of 0.25 mg/L. Assuming that the Vd of morphine in this patient is 200 L and the half-life is 3 h, how much morphine did the patient inject 6 h earlier? (A) 25 mg (B) 50 mg (C) 100 mg (D) 200 mg (E) Not enough data to predict 10. Gentamicin, an aminoglycoside antibiotic, is sometimes given in intermittent intravenous bolus doses of 100 mg 3 times a day to achieve target peak plasma concentrations of about 5 mg/L. Gentamicin’s clearance (normally 5.4 L/h/70 kg) is almost entirely by glomerular filtration. Your patient, however, is found to have a creatinine clearance one third of normal. What should your modified dosage regimen for this patient be? (A) 20 mg 3 times a day (B) 33 mg 3 times a day (C) 72 mg 3 times a day (D) 100 mg 2 times a day (E) 150 mg 2 times a day DRUG METABOLOSIM 1. Drug metabolism in humans usually results in a product that is (A) Less lipid soluble than the original drug (B) More likely to distribute intracellularly (C) More likely to be reabsorbed by kidney tubules (D) More lipid soluble than the original drug (E) Less water soluble than the original drug 2. If therapy with multiple drugs causes induction of drug metabolism in your depressed patient, it will (A) Be associated with increased smooth endoplasmic reticulum (B) Be associated with increased rough endoplasmic reticulum (C) Be associated with decreased enzymes in the soluble cytoplasmic fraction (D) Require 3–4 months to reach completion (E) Be irreversible 3. Which of the following factors is likely to increase the duration of action of a drug that is metabolized by CYP3A4 in the liver? (A) Chronic administration of rifampin during therapy with the drug in question (B) Chronic therapy with amiodarone (C) Displacement from tissue-binding sites by another drug (D) Increased cardiac output (E) Chronic administration of carbamazepine 4. Reports of cardiac arrhythmias caused by unusually high blood levels of 2 antihistamines, terfenadine and astemizole, led to their removal from the market. Which of the following best explains these effects? (A) Concomitant treatment with rifampin (B) Use of these drugs by chronic alcoholics (C) Use of these drugs by chronic smokers (D) Treatment of these patients with ketoconazole, an azole antifungal agent 5. Which of the following agents, when used in combination with other anti-HIV drugs, permits dose reductions? (A) Cimetidine (B) Efavirenz (C) Ketoconazole (D) Procainamide (E) Quinidine (F) Ritonavir (G) Succinylcholine (H) Verapamil 6. Which of the following drugs may inhibit the hepatic microsomal P450 responsible for warfarin metabolism? (A) Amiodarone (B) Ethanol (C) Phenobarbital (D) Procainamide (E) Rifampin 7. Which of the following drugs, if used chronically, is most likely to increase the toxicity of acetaminophen? (A) Cimetidine (B) Ethanol (C) Ketoconazole (D) Procainamide (E) Quinidine (F) Ritonavir (G) Succinylcholine (H) Verapamil 8. Which of the following drugs has higher first-pass metabolism in men than in women? (A) Cimetidine (B) Ethanol (C) Ketoconazole (D) Procainamide (E) Quinidine (F) Ritonavir (G) Succinylcholine (H) Verapamil 9. Which of the following drugs is an established inhibitor of P-glycoprotein (P-gp) drug transporters? (A) Cimetidine (B) Ethanol (C) Ketoconazole (D) Procainamide (E) Quinidine (F) Ritonavir (G) Succinylcholine (H) Verapamil 10. Which of the following cytochrome isoforms is responsible for metabolizing the largest number of drugs? (A) CYP1A2 (B) CYP2C9 (C) CYP2C19 (D) CYP2D6 (E) CYP3A4 PHARMACOGENOMICS 1. A 59-year-old man with acute coronary syndrome is admitted to the hospital for emergency percutaneous insertion of a coronary stent. Which of the following drugs might cause unexpected results based on the patient’s CYP2C19 genotype? (A) Clopidogrel (B) Codeine (C) Prasugrel (D) Ticagrelor (E) Warfarin 2. A 62-year-old woman with advanced colon cancer is treated with intravenous 5-fluorouracil. Within a few days, she develops severe diarrhea, and within a week, she shows severe neutropenia. Which of the following polymorphisms is most likely to be responsible? (A) CYP2D6 *1x3 (B) CYP2C19*2 (C) CYP2C9*3 (D) DYPD *2A (E) UGT1A1*28 3. A 38-year-old man is being treated for HIV-induced acquired immunodeficiency syndrome (AIDS). When abacavir therapy is begun, he develops a severe skin rash. Which of the following pharmacogenomic diagnoses might explain this skin rash? (A) CYP2D6 *3 (PM) (B) CYP3A5 *3 (PM) (C) HLA-B *57:01 (EM) (D) SLCO1B1*5 (PM) 4. A college student volunteers to have his genome decoded as part of a population-wide study of polymorphisms. He receives a call from the principal investigator informing him that his genome unexpectedly contains an important single nucleotide polymorphism. Which of the following polymorphisms is associated with risk of hemolysis and increased resistance to malaria? (A) CYP2D6 *3 (B) CYP2D19*2 (C) TPMT *2 (D) UGT1A1*28 (E) G6PD-(A)–Canton 5. A 7-year-old child is brought to the emergency department in coma with cyanosis. Her mother states that the girl was given codeine with acetaminophen because of severe bruising after a fall. Shortly after the first dose, the child became unresponsive and “turned blue.” Which of the following alleles might be responsible for this presentation? (A) CYP2D6 *1x3 (B) CYP2C19*2 (C) CYP2C9*3 (D) DYPD*2A (E) UGT1A1*28 PART 2 Autonomic Drugs INTRODUCTION TO AUTONOMIC PHARMACOLOGY 1. A 3-year-old child has been admitted to the emergency department having swallowed the contents of 2 bottles of a nasal decongestant. The active ingredient of the medication is a potent, selective α-adrenoceptor agonist drug. Which of the following is a sign of α-receptor activation that may occur in this patient? (A) Bronchodilation (B) Cardiac acceleration (tachycardia) (C) Pupillary dilation (mydriasis) (D) Renin release from the kidneys (E) Vasodilation of the blood vessels of the skin 2. Mr Green is a 60-year-old man with poorly controlled hypertension of 170/110 mm Hg. He is to receive minoxidil. Minoxidil is a powerful arteriolar vasodilator that does not act on autonomic receptors. Which of the following effects will be observed if no other drugs are used? (A) Tachycardia and increased cardiac contractility (B) Tachycardia and decreased cardiac output (C) Decreased mean arterial pressure and decreased cardiac contractility (D) Decreased mean arterial pressure and increased salt and water excretion by the kidney (E) No change in mean arterial pressure and decreased cardiac Contractility 3. Full activation of the parasympathetic nervous system is likely to produce which of the following effects? (A) Bronchodilation (B) Decreased intestinal motility (C) Increased thermoregulatory sweating (D) Increased pupillary constrictor tone (miosis) (E) Increased heart rate (tachycardia) Questions 4–5. For these questions, use the accompanying diagram. Assume that the diagram can represent either the sympathetic or the parasympathetic system. 4. Assuming the structure is part of the thoracolumbar system, norepinephrine acts at which of the following sites in the diagram? (A) Sites 1 and 2 (B) Sites 3 and 4 (C) Sites 5 and 6 5. If the effector cell in the diagram is a pupillary constrictor smooth muscle cell, which of the following receptor types is denoted by structure 6? (A) Alpha1 adrenoceptor (B) Beta2 adrenoceptor (C) M3 cholinoceptor (D) Ng cholinoceptor 6. Nicotinic receptor sites do not include which one of the following sites? (A) Bronchial smooth muscle (B) Adrenal medullary cells (C) Parasympathetic ganglia (D) Skeletal muscle end plates (E) Sympathetic ganglia 7. Several children at a summer camp were hospitalized with symptoms thought to be due to ingestion of food containing botulinum toxin. Which one of the following signs or symptoms is consistent with the diagnosis of botulinum poisoning? (A) Bronchospasm (B) Cycloplegia (C) Diarrhea (D) Skeletal muscle spasms (E) Hyperventilation 8. Which one of the following is the primary neurotransmitter agent normally released in the sinoatrial node of the heart in response to a blood pressure increase? (A) Acetylcholine (B) Dopamine (C) Epinephrine (D) Glutamate (E) Norepinephrine Questions 9–10. Assume that the diagram below represents a sympathetic postganglionic nerve ending. 9. Which of the following blocks the carrier represented by “z” in the diagram? (A) Amphetamine (B) Botulinum toxin (C) Cocaine (D) Hemicholinium (E) Reserpine 10. Which of the following inhibits the carrier denoted “y” in the diagram? (A) Cocaine (B) Dopamine (C) Hemicholinium (D) Reserpine (E) Vesamicol Cholinoceptor-Activating & Cholinesterase-Inhibiting Drugs 1. A 30-year-old woman undergoes abdominal surgery. In spite of minimal tissue damage, complete ileus (absence of bowel motility) follows, and she complains of severe bloating. She also finds it difficult to urinate. Mild cholinomimetic stimulation with bethanechol or neostigmine is often effective in relieving these complications of surgery. Neostigmine and bethanechol in moderate doses have significantly different effects on which one of the following? (A) Gastric secretory cells (B) Vascular endothelium (C) Salivary glands (D) Sweat glands (E) Ureteral tone 2. Parathion has which one of the following characteristics? (A) It is inactivated by conversion to paraoxon (B) It is less toxic to humans than malathion (C) It is more persistent in the environment than DDT (D) It is poorly absorbed through skin and lungs (E) If treated early, its toxicity may be partly reversed by pralidoxime 3. Ms Brown has been treated for myasthenia gravis for several years. She reports to the emergency department complaining of recent onset of weakness of her hands, diplopia, and difficulty swallowing. She may be suffering from a change in response to her myasthenia therapy, that is, a cholinergic or a myasthenic crisis. Which of the following is the best drug for distinguishing between myasthenic crisis (insufficient therapy) and cholinergic crisis (excessive therapy)? (A) Atropine (B) Edrophonium (C) Physostigmine (D) Pralidoxime (E) Pyridostigmine 4. A crop duster pilot has been accidentally exposed to a high concentration of a highly toxic agricultural organophosphate insecticide. If untreated, the cause of death from such exposure would probably be (A) Cardiac arrhythmia (B) Gastrointestinal bleeding (C) Heart failure (D) Hypotension (E) Respiratory failure 5. Mr Green has just been diagnosed with dysautonomia (chronic idiopathic autonomic insufficiency). You are considering different therapies for his disease. Pyridostigmine and neostigmine may cause which one of the following in this patient? (A) Bronchodilation (B) Cycloplegia (C) Diarrhea (D) Irreversible inhibition of acetylcholinesterase (E) Reduced gastric acid secretion 6. Parasympathetic nerve stimulation and a slow infusion of bethanechol will each (A) Cause ganglion cell depolarization (B) Cause skeletal muscle end plate depolarization (C) Cause vasodilation (D) Increase bladder tone (E) Increase heart rate 7. Actions and clinical uses of muscarinic cholinoceptor agonists include which one of the following? (A) Bronchodilation (treatment of asthma) (B) Miosis (treatment of glaucoma) (C) Decreased gastrointestinal motility (treatment of diarrhea) (D) Decreased neuromuscular transmission and relaxation of skeletal muscle (during surgical anesthesia) (E) Increased sweating (treatment of fever) 8. Which of the following is a direct-acting cholinomimetic that is lipid-soluble and is used to facilitate smoking cessation? (A) Acetylcholine (B) Bethanechol (C) Neostigmine (D) Physostigmine (E) Varenicline 9. A 3-year-old child is admitted to the emergency department after taking a drug from her parents’ medicine cabinet. The signs suggest that the drug is an indirect-acting cholinomimetic with little or no CNS effect and a duration of action of about 2–4 h. Which of the following is the most likely cause of these effects? (A) Acetylcholine (B) Bethanechol (C) Neostigmine (D) Physostigmine (E) Pilocarpine 10. Which of the following is the primary second-messenger process in the contraction of the ciliary muscle when focusing on near objects? (A) cAMP (cyclic adenosine monophosphate) (B) DAG (diacylglycerol) (C) Depolarizing influx of sodium ions via a channel (D) IP3 (inositol 1,4,5-trisphosphate) (E) NO (nitric oxide) Cholinoceptor Blockers &Cholinesterase Regenerators 1. A 27-year old compulsive drug user injected a drug he thought was methamphetamine, but he has not developed any signs of methamphetamine action. He has been admitted to the emergency department and antimuscarinic drug overdose is suspected. Probable signs of atropine overdose include which one of the following? (A) Gastrointestinal smooth muscle cramping (B) Increased heart rate (C) Increased gastric secretion (D) Pupillary constriction (E) Urinary frequency 2. Which of the following is the most dangerous effect of belladonna alkaloids in infants and toddlers? (A) Dehydration (B) Hallucinations (C) Hypertension (D) Hyperthermia (E) Intraventricular heart block 3. Which one of the following can be blocked by atropine? (A) Decreased blood pressure caused by hexamethonium (B) Increased blood pressure caused by nicotine (C) Increased skeletal muscle strength caused by neostigmine (D) Tachycardia caused by exercise (E) Sweating caused by exercise Questions 4–5. Two new synthetic drugs (X and Y) are to be studied for their cardiovascular effects. The drugs are given to three anesthetized animals while the blood pressure is recorded. The first animal has received no pretreatment (control), the second has received an effective dose of a long-acting ganglion blocker, and the third has received an effective dose of a long-acting muscarinic antagonist. 4. Drug X caused a 50 mm Hg rise in mean blood pressure in the control animal, no blood pressure change in the ganglionblocked animal, and a 75 mm mean blood pressure rise in the atropine-pretreated animal. Drug X is probably a drug similar to (A) Acetylcholine (B) Atropine (C) Epinephrine (D) Hexamethonium (E) Nicotine 5. The net changes in heart rate induced by drug Y in these experiments are shown in the following graph. Drug Y is probably a drug similar to (A) Acetylcholine (B) Edrophonium (C) Hexamethonium (D) Nicotine (E) Pralidoxime 6. A 30-year-old man has been treated with several autonomic drugs for 4 weeks. He is now admitted to the emergency department showing signs of drug toxicity. Which of the following signs would distinguish between an overdose of a ganglion blocker versus a muscarinic blocker? (A) Cycloplegia (B) Dry skin in a warm environment (C) Miosis (D) Postural hypotension (E) Tachycardia 7. Which of the following is an accepted therapeutic indication for the use of antimuscarinic drugs? (A) Atrial fibrillation (B) Botulinum poisoning (C) Chronic obstructive pulmonary disease (COPD) (D) Glaucoma (E) Postoperative urinary retention 8. Which of the following is an expected effect of a therapeutic dose of an antimuscarinic drug? (A) Decreased cAMP (cyclic adenosine monophosphate) in cardiac muscle (B) Decreased DAG (diacylglycerol) in salivary gland tissue (C) Increased IP3 (inositol trisphosphate) in intestinal smooth muscle (D) Increased potassium efflux from smooth muscle (E) Increased sodium influx into the skeletal muscle end plate 9. Which one of the following drugs causes vasodilation that can be blocked by atropine? (A) Benztropine (B) Bethanechol (C) Botulinum toxin (D) Cyclopentolate (E) Edrophonium (F) Neostigmine (G) Pralidoxime 10. Which one of the following drugs has a very high affinity for the phosphorus atom in parathion and is often used to treat life-threatening insecticide toxicity? (A) Atropine (B) Benztropine (C) Bethanechol (D) Botulinum (E) Cyclopentolate (F) Neostigmine (G) Pralidoxime Sympathomimetics Questions 1 and 2. A 7-year-old boy with a previous history of bee sting allergy is brought to the emergency department after being stung by 3 bees. 1. Which of the following are probable signs of the anaphylactic reaction to bee stings? (A) Bronchodilation, tachycardia, hypertension, vomiting, diarrhea (B) Bronchospasm, tachycardia, hypotension, laryngeal edema (C) Diarrhea, bradycardia, vomiting (D) Laryngeal edema, bradycardia, hypotension, diarrhea (E) Miosis, tachycardia, vomiting, diarrhea 2. If this child has signs of anaphylaxis, what is the treatment of choice? (A) Diphenhydramine (an antihistamine) (B) Ephedrine (C) Epinephrine (D) Isoproterenol (E) Methylprednisolone (a corticosteroid) 3. A 65-year-old woman with impaired renal function and a necrotic ulcer in the sole of her right foot is admitted to the ward from the emergency department. She has long-standing type 2 diabetes mellitus and you wish to examine her retinas for possible vascular changes. Which of the following drugs is a good choice when pupillary dilation—but not cycloplegia— is desired? (A) Isoproterenol (B) Norepinephrine (C) Phenylephrine (D) Pilocarpine (E) Tropicamide 4. A 60-year-old immigrant from Latin America was told she had hypertension and should be taking antihypertensive medication. She decides to take an herbal medication from an online “holistic pharmacy.” One week after starting the medication, she is found unconscious in her apartment. In the emergency department, her blood pressure is 50/0 mm Hg and heart rate is 40 bpm. Respirations are 20/min; pupils are slightly constricted. Bowel sounds are present. Which of the following would be the most effective cardiovascular stimulant? (A) Amphetamine (B) Clonidine (C) Isoproterenol (D) Norepinephrine (E) Tyramine 5. A group of volunteers are involved in a phase 1 clinical trial of a new autonomic drug. When administered by intravenous bolus, the blood pressure increases. When given orally for 1 week, the blood pressure decreases. Which of the following standard agents does the new drug most resemble? (A) Atropine (B) Clonidine (C) Phentolamine (an α blocker) (D) Phenylephrine (E) Propranolol (a β blocker) 6. Your 30-year-old patient has moderately severe new onset asthma, and you prescribe a highly selective β2 agonist inhaler to be used when needed. In considering the possible drug effects in this patient, you would note that β2 stimulants frequently cause (A) Direct stimulation of renin release (B) Hypoglycemia (C) Itching due to increased cGMP (cyclic guanine monophosphate) in mast cells (D) Skeletal muscle tremor (E) Vasodilation in the skin 7. Mr Green, a 54-year-old banker, had a cardiac transplant 6 months ago. His current blood pressure is 120/70 mm Hg and heart rate is 100 bpm. Which of the following drugs would have the least effect on Mr Green's heart rate? (A) Albuterol (B) Epinephrine (C) Isoproterenol (D) Norepinephrine (E) Phenylephrine Questions 8 and 9. Several new drugs with autonomic actions were studied in preclinical trials in animals. Autonomic drugs X and Y were given in moderate doses as intravenous boluses. The systolic and diastolic blood pressures changed as shown in the diagram below. 8. Which of the following drugs most resembles drug X? (A) Atropine (B) Bethanechol (C) Epinephrine (D) Isoproterenol (E) Phenylephrine 9. Which of the following most resembles drug Y? (A) Atropine (B) Bethanechol (C) Epinephrine (D) Isoproterenol (E) Phenylephrine 10. A new drug was given by subcutaneous injection to 25 normal subjects in a phase 1 clinical trial. The cardiovascular effects are summarized in the table below. Which of the following drugs does the new experimental agent most resemble? (A) Atropine (B) Epinephrine (C) Isoproterenol (D) Phenylephrine (E) Physostigmine Adrenoceptor Blockers 1.A patient is to receive epinephrine. She has previously received an adrenoceptor-blocking agent. Which of the following effects of epinephrine would be blocked by phentolamine but not by metoprolol? (A) Cardiac stimulation (B) Increase of cAMP (cyclic adenosine monophosphate) in fat (C) Mydriasis (D) Relaxation of bronchial smooth muscle (E) Relaxation of the uterus 2. Clinical studies have shown that adrenoceptor blockers have many useful effects in patients. However, a number of drug toxicities have been documented. Adverse effects that limit the use of adrenoceptor blockers include which one of the following? (A) Bronchoconstriction from α-blocking agents (B) Acute heart failure exacerbation from β blockers (C) Impaired blood sugar response with α blockers (D) Increased intraocular pressure with β blockers (E) Sleep disturbances from α-blocking drugs Four new synthetic drugs (designated W, X, Y, and Z) are to be studied for their cardiovascular effects. They are given to 4 anesthetized animals while the heart rate is recorded. The first animal has received no pretreatment (control); the second has received an effective dose of hexamethonium; the third has received an effective dose of atropine; and the fourth has received an effective dose of phenoxybenzamine. The net changes induced by W, X, Y, and Z in the animals are described in the following questions. 3. Drug W increased heart rate in the control animal, the atropine-pretreated animal, and the phenoxybenzaminepretreated animal. However, drug W had no effect on heart rate in the hexamethonium-pretreated animal. Drug W is probably a drug similar to (A) Acetylcholine (B) Edrophonium (C) Isoproterenol (D) Nicotine (E) Norepinephrine 4. Drug X had the effects shown in the table below. No pretreatment ↓ Hexamethonium ↑ Atropine ↑ Phenoxybenzamine ↑ Drug X is probably a drug similar to (A) Acetylcholine (B) Albuterol (C) Edrophonium (D) Isoproterenol (E) Norepinephrine 5. Drug Y had the effects shown in the table below. No pretreatment ↑ Hexamethonium ↑ Atropine ↑ Phenoxybenzamine ↑ Drug Y is probably a drug similar to (A) Acetylcholine (B) Edrophonium (C) Isoproterenol (D) Norepinephrine (E) Prazosin 6. The results of the test of drug Z are shown in the graph. Drug Z is probably a drug similar to (A) Acetylcholine (B) Edrophonium (C) Isoproterenol (D) Norepinephrine (E) Pralidoxime 7. When given to a patient, phentolamine blocks which one of the following? (A) Bradycardia induced by phenylephrine (B) Bronchodilation induced by epinephrine (C) Increased cardiac contractile force induced by norepinephrine (D) Miosis induced by acetylcholine (E) Vasodilation induced by isoproterenol 8. Your 75-year-old patient with angina and glaucoma is to receive a β-blocking drug. Which of the following statements is most correct regarding β-blocking drugs? (A) Esmolol’s pharmacokinetics are compatible with chronic topical use (B) Metoprolol blocks β2 receptors selectively (C) Nadolol lacks β2-blocking action (D) Pindolol is a β antagonist with high membrane-stabilizing (local anesthetic) activity (E) Timolol lacks the local anesthetic effects of propranolol 9. A 56-year-old man has hypertension and an enlarged prostate, which biopsy shows to be benign prostatic hyperplasia. He complains of urinary retention. Which of the following drugs would be the most appropriate initial therapy? (A) Albuterol (B) Atenolol (C) Metoprolol (D) Prazosin (E) Timolol 10. A new drug was administered to an anesthetized animal with the results shown here. A large dose of epinephrine (epi) was administered before and after the new agent for comparison. Which of the following agents does the new drug most closely resemble? (A) Atenolol (B) Atropine (C) Labetalol (D) Phenoxybenzamine (E) Propranolol 11. DRUGS USED IN HYPERTENSION 1. A 32-year-old woman with hypertension wishes to become pregnant. Her physician informs her that she will have to switch to another antihypertensive drug. Which of the following drugs is absolutely contraindicated in pregnancy? (A) Atenolol (B) Losartan (C) Methyldopa (D) Nifedipine (E) Propranolol 2. A patient is admitted to the emergency department with severe tachycardia after a drug overdose. His family reports that he has been depressed about his hypertension. Which one of the following drugs increases the heart rate in a dosedependent manner? (A) Captopril (B) Hydrochlorothiazide (C) Losartan (D) Minoxidil (E) Verapamil 3. Which one of the following is characteristic of nifedipine treatment in patients with essential hypertension? (A) Competitively blocks angiotensin II at its receptor (B) Decreases calcium efflux from skeletal muscle (C) Decreases renin concentration in the blood (D) Decreases calcium influx into smooth muscle (E) Decreases calcium flux into the urine 4. A 73-year-old man with a history of a recent change in his treatment for moderately severe hypertension is brought to the emergency department because of a fall at home. Which of the following drug groups is most likely to cause postural hypotension and thus an increased risk of falls? (A) ACE inhibitors (B) Alpha1-selective receptor blockers (C) Arteriolar dilators (D) Beta1-selective receptor blockers (E) Nonselective β blockers 5. A significant number of patients started on ACE inhibitor therapy for hypertension are intolerant and must be switched to a different class of drug. What is the most common manifestation of this intolerance? (A) Angioedema (B) Glaucoma (C) Headache (D) Incessant cough (E) Ventricular arrhythmias 6. Which one of the following is a significant unwanted effect of the drug named? (A) Constipation with verapamil (B) Heart failure with hydralazine (C) Hemolytic anemia with atenolol (D) Hypokalemia with aliskiren (E) Lupus-like syndrome with hydrochlorothiazide 7. Comparison of prazosin with atenolol shows that (A) Both decrease heart rate (B) Both increase cardiac output (C) Both increase renin secretion (D) Both increase sympathetic outflow from the CNS (E) Both produce orthostatic hypotension 8. A patient with hypertension and angina is referred for treatment. Metoprolol and verapamil are among the drugs considered. Both metoprolol and verapamil are associated with which one of the following? (A) Diarrhea (B) Hypoglycemia (C) Increased PR interval (D) Tachycardia (E) Thyrotoxicosis 9. A 45-year-old man is brought to the emergency department with mental obtundation. He is found to have a blood pressure of 220/160 and retinal hemorrhages. Which one of the following is used in severe hypertensive emergencies, is shortacting, acts on a G protein-coupled receptor, and must be given by intravenous infusion? (A) Aliskiren (B) Captopril (C) Fenoldopam (D) Hydralazine (E) Losartan (F) Metoprolol (G) Nitroprusside (H) Prazosin (I) Propranolol 10. Which of the following is very short-acting and acts by releasing nitric oxide? (A) Atenolol (B) Captopril (C) Diltiazem (D) Fenoldopam (E) Hydrochlorothiazide (F) Losartan (G) Minoxidil (H) Nitroprusside (I) Prazosin Drugs Used in the Treatment of Angina Pectoris Questions 1–4. A 60-year-old man presents to his primary care physician with a complaint of severe chest pain when he walks uphill to his home in cold weather. The pain disappears when he rests. After evaluation and discussion of treatment options, a decision is made to treat him with nitroglycerin. 1. Which of the following is a common direct or reflex effect of nitroglycerin? (A) Decreased heart rate (B) Decreased venous capacitance (C) Increased afterload (D) Increased cardiac force (E) Increased diastolic myocardial fiber tension 2. In advising the patient about the adverse effects he may notice, you point out that nitroglycerin in moderate doses often produces certain symptoms. Which of the following effects might occur due to the mechanism listed? (A) Constipation (B) Dizziness due to reduced cardiac force of contraction (C) Diuresis due to sympathetic discharge (D) Headache due to meningeal vasodilation (E) Hypertension due to reflex tachycardia 3. One year later, the patient returns complaining that his nitroglycerin works well when he takes it for an acute attack but that he is now having more frequent attacks and would like something to prevent them. Useful drugs for the prophylaxis of angina of effort include (A) Amyl nitrite (B) Esmolol (C) Sublingual isosorbide dinitrate (D) Sublingual nitroglycerin (E) Verapamil 4. If a β blocker were to be used for prophylaxis in this patient, what is the most probable mechanism of action in angina? (A) Block of exercise-induced tachycardia (B) Decreased end-diastolic ventricular volume (C) Increased double product (D) Increased cardiac force (E) Decreased ventricular ejection time 5. A new 60-year-old patient presents to the medical clinic with hypertension and angina. He is 1.8 meters tall with a waist measurement of 1.1 m. Weight is 97 kg. Blood pressure is 150/95 and pulse 85. In considering adverse effects of possible drugs for these conditions, you note that an adverse effect that nitroglycerin and prazosin have in common is (A) Bradycardia (B) Impaired sexual function (C) Lupus erythematosus syndrome (D) Orthostatic hypotension (E) Weight gain 6. A man is admitted to the emergency department with a brownish cyanotic appearance, marked shortness of breath, and hypotension. Which of the following is most likely to cause methemoglobinemia? (A) Amyl nitrite (B) Isosorbide dinitrate (C) Isosorbide mononitrate (D) Nitroglycerin (E) Sodium cyanide 7. Another patient is admitted to the emergency department after a drug overdose. He is noted to have hypotension and severe bradycardia. He has been receiving therapy for hypertension and angina. Which of the following drugs in high doses causes bradycardia? (A) Amlodipine (B) Isosorbide dinitrate (C) Nitroglycerin (D) Prazosin (E) Verapamil 8. A 45-year-old woman with hyperlipidemia and frequent migraine headaches develops angina of effort. Which of the following is relatively contraindicated because of her migraines? (A) Amlodipine (B) Diltiazem (C) Metoprolol (D) Nitroglycerin (E) Verapamil 9. When nitrates are used in combination with other drugs for the treatment of angina, which one of the following combinations results in additive effects on the variable specified? (A) Beta blockers and nitrates on end-diastolic cardiac size (B) Beta blockers and nitrates on heart rate (C) Beta blockers and nitrates on venous tone (D) Calcium channel blockers and β blockers on cardiac force (E) Calcium channel blockers and nitrates on heart rate 10. Certain drugs can cause severe hypotension when combined with nitrates. Which of the following interacts with nitroglycerin by inhibiting the metabolism of cGMP? (A) Atenolol (B) Hydralazine (C) Isosorbide mononitrate (D) Nifedipine (E) Ranolazine (F) Sildenafil (G) Terbutaline DRUGD USED IN HEART FAILURE A 73-year-old man with an inadequate response to other drugs is to receive digoxin for chronic heart failure. He is in normal sinus rhythm with a heart rate of 88 and blood pressure of 135/85 mm Hg. 1. Which of the following is the best-documented mechanism of beneficial action of cardiac glycosides? (A) A decrease in calcium uptake by the sarcoplasmic reticulum (B) An increase in ATP synthesis (C) A modification of the actin molecule (D) An increase in systolic cytoplasmic calcium levels (E) A block of cardiac β adrenoceptors 2. After your patient has been receiving digoxin for 3 wk, he presents to the emergency department with an arrhythmia. Which one of the following is most likely to contribute to the arrhythmogenic effect of digoxin? (A) Increased parasympathetic discharge (B) Increased intracellular calcium (C) Decreased sympathetic discharge (D) Decreased intracellular ATP (E) Increased extracellular potassium 3. A patient who has been taking digoxin for several years for atrial fibrillation and chronic heart failure is about to receive atropine for another condition. A common effect of digoxin (at therapeutic blood levels) that can be almost entirely blocked by atropine is (A) Decreased appetite (B) Headaches (C) Increased atrial contractility (D) Increased PR interval on ECG (E) Tachycardia 4. A 65-year-old woman has been admitted to the coronary care unit with a left ventricular myocardial infarction. She develops acute severe heart failure with marked pulmonary edema, but no evidence of peripheral edema. Which one of the following drugs would be most useful? (A) Digoxin (B) Furosemide (C) Minoxidil (D) Propranolol (E) Spironolactone 5. A 72-year-old woman has long-standing heart failure. Which one of the following drugs has been shown to reduce mortality in chronic heart failure? (A) Atenolol (B) Digoxin (C) Dobutamine (D) Furosemide (E) Spironolactone 6. Which row in the following table correctly shows the major effects of full therapeutic doses of digoxin on the AV node and the ECG? Row AV Refractory Period QT Interval T Wave (A) Increased Increased Upright (B) Increased Decreased Inverted (C) Decreased Increased Upright (D) Decreased Decreased Upright (E) Decreased Increased Inverted 7. Which one of the following drugs is associated with clinically useful or physiologically important positive inotropic effect? (A) Captopril (B) Dobutamine (C) Enalapril (D) Losartan (E) Nesiritide 8. A 68-year-old man with a history of chronic heart failure goes on vacation and abandons his low-salt diet. Three days later, he develops severe shortness of breath and is admitted to the local hospital emergency department with significant pulmonary edema. The first-line drug of choice in most cases of acute decompensation in patients with chronic heart failure is (A) Atenolol (B) Captopril (C) Carvedilol (D) Digoxin (E) Diltiazem (F) Dobutamine (G) Enalapril (H) Furosemide (I) Metoprolol (J) Spironolactone 9. Which of the following has been shown to prolong life in patients with chronic congestive failure in spite of having a negative inotropic effect on cardiac contractility? (A) Carvedilol (B) Digoxin (C) Dobutamine (D) Enalapril (E) Furosemide 10. A 5-year-old child was vomiting and was brought to the emergency department with sinus arrest and a ventricular rate of 35 bpm. An empty bottle of his uncle’s digoxin was found where he was playing. Which of the following is the drug of choice in treating a severe overdose of digoxin? (A) Digoxin antibodies (B) Lidocaine infusion (C) Magnesium infusion (D) Phenytoin by mouth (E) Potassium by mouth Antiarrhythmic Drugs 1. A 76-year-old patient with rheumatoid arthritis and chronic heart disease is being considered for treatment with procainamide. She is already receiving digoxin, hydrochlorothiazide, and potassium supplements for her cardiac condition. 1. In deciding on a treatment regimen with procainamide for this patient, which of the following statements is most correct? (A) A possible drug interaction with digoxin suggests that digoxin blood levels should be obtained before and after starting procainamide (B) Hyperkalemia should be avoided to reduce the likelihood of procainamide toxicity (C) Procainamide cannot be used if the patient has asthma because it has a β-blocking effect (D) Procainamide cannot be used if the patient has angina because it has a β-agonist effect (E) Procainamide is not active by the oral route 2. If this patient should take an overdose and manifest severe acute procainamide toxicity with markedly prolonged QRS, which of the following should be given immediately? (A) A calcium chelator such as EDTA (B) Digitalis (C) Nitroprusside (D) Potassium chloride (E) Sodium lactate 3. A 57-year-old man is admitted to the emergency department with chest pain and a fast irregular heart rhythm. The ECG shows an inferior myocardial infarction and ventricular tachycardia. Lidocaine is ordered. When used as an antiarrhythmic drug, lidocaine typically (A) Increases action potential duration (B) Increases contractility (C) Increases PR interval (D) Reduces abnormal automaticity (E) Reduces resting potential 4. A 36-year-old woman with a history of poorly controlled thyrotoxicosis has recurrent episodes of tachycardia with severe shortness of breath. When she is admitted to the emergency department with one of these episodes, which of the following drugs would be most suitable? (A) Amiodarone (B) Disopyramide (C) Esmolol (D) Quinidine (E) Verapamil 5. A 16-year-old girl has paroxysmal attacks of rapid heart rate with palpitations and shortness of breath. These episodes occasionally terminate spontaneously but often require a visit to the emergency department of the local hospital. Her ECG during these episodes reveals an AV nodal tachycardia. The antiarrhythmic of choice in most cases of acute AV nodal tachycardia is (A) Adenosine (B) Amiodarone (C) Flecainide (D) Propranolol (E) Verapamil 6. A 55-year-old man is admitted to the emergency department and is found to have an abnormal ECG. Overdose of an antiarrhythmic drug is considered. Which of the following drugs is correctly paired with its ECG effects? (A) Quinidine: Increased PR and decreased QT intervals (B) Flecainide: Increased PR, QRS, and QT intervals (C) Verapamil: Increased PR interval (D) Lidocaine: Decreased QRS and PR interval (E) Metoprolol: Increased QRS duration 7. A 60-year-old man comes to the emergency department with severe chest pain. ECG reveals ventricular tachycardia with occasional normal sinus beats, and ST-segment changes suggestive of ischemia. A diagnosis of myocardial infarction is made, and the man is admitted to the cardiac intensive care unit. His arrhythmia should be treated immediately with (A) Adenosine (B) Digoxin (C) Lidocaine (D) Quinidine (E) Verapamil 8. Which of the following drugs slows conduction through the AV node and has its primary action directly on L-type calcium channels? (A) Adenosine (B) Amiodarone (C) Diltiazem (D) Esmolol (E) Flecainide (F) Lidocaine (G) Mexiletine (H) Procainamide (I) Quinidine 9. When working in outlying areas, this 62-year-old rancher is away from his house for 12–14 h at a time. He has an arrhythmia that requires chronic therapy. Which of the following has the longest half-life of all antiarrhythmic drugs? (A) Adenosine (B) Amiodarone (C) Disopyramide (D) Esmolol (E) Flecainide (F) Lidocaine (G) Mexiletine (H) Procainamide (I) Quinidine (J) Verapamil 10.A drug was tested in the electrophysiology laboratory to determine its effects on the cardiac action potential in normal ventricular cells. The results are shown in the diagram. Which of the following drugs does this agent most resemble? (A) Adenosine (B) Flecainide (C) Mexiletine (D) Procainamide (E) Verapamil Diuretics & Other Drugs That Act on the Kidney 1. A 70-year-old retired businessman is admitted with a history of recurrent heart failure and metabolic derangements. He has marked peripheral edema and metabolic alkalosis. Which of the following drugs is most appropriate for the treatment of his edema? (A) Acetazolamide (B) Digoxin (C) Dobutamine (D) Eplerenone (E) Hydrochlorothiazide 2. A 50-year-old man has a history of frequent episodes of renal colic with calcium-containing renal stones. A careful workup indicates that he has a defect in proximal tubular calcium reabsorption, which results in high concentrations of calcium salts in the tubular urine. The most useful diuretic agent in the treatment of recurrent calcium stones is (A) Chlorthalidone (B) Diazoxide (C) Ethacrynic acid (D) Mannitol (E) Spironolactone 3. Which of the following is an important effect of chronic therapy with loop diuretics? (A) Decreased urinary excretion of calcium (B) Elevation of blood pressure (C) Elevation of pulmonary vascular pressure (D) Metabolic alkalosis (E) Teratogenic action in pregnancy 4. Which drug is correctly associated with its actions in the following table? (+ indicates increase and – indicates decrease.) Choice Drug UrineNa+ UrineK+ Metabolicchange A Acetazolamide +++ + Alkalosis B Furosemide ++ - Alkalosis C Hydrochlorothiazide + + + Acidosis D Spironolactone + – Acidosis E Mannitol – ++ Alkalosis 5. Which of the following diuretics would be most useful in the acute treatment of a comatose patient with traumatic brain injury and cerebral edema? (A) Acetazolamide (B) Amiloride (C) Chlorthalidone (D) Furosemide (E) Mannitol 6. A 62-year-old man with advanced prostate cancer is admitted to the emergency department with mental obtundation. An electrolyte panel shows a serum calcium of 16.5 (normal 8.5–10.5 mg/dL). Which of the following therapies would be most useful in the management of severe hypercalcemia? (A) Acetazolamide plus saline infusion (B) Furosemide plus saline infusion (C) Hydrochlorothiazide plus saline infusion (D) Mannitol plus saline infusion (E) Spironolactone plus saline infusion 7. A 60-year-old patient complains of paresthesias and occasional nausea associated with one of her drugs. She is found to have hyperchloremic metabolic acidosis. She is probably taking (A) Acetazolamide for glaucoma (B) Amiloride for edema associated with aldosteronism (C) Furosemide for severe hypertension and heart failure (D) Hydrochlorothiazide for hypertension (E) Mannitol for cerebral edema 8. A 70-year-old woman is admitted to the emergency department because of a “fainting spell” at home. She appears to have suffered no trauma from her fall, but her blood pressure is 120/60 when lying down and 60/20 when she sits up. Neurologic examination and an ECG are within normal limits when she is lying down. Questioning reveals that she has recently started taking “water pills” (diuretics) for a heart condition. Which of the following drugs is the most likely cause of her fainting spell? (A) Acetazolamide (B) Amiloride (C) Furosemide (D) Hydrochlorothiazide (E) Spironolactone 9. A 58-year-old woman with lung cancer has abnormally low serum osmolality and hyponatremia. A drug that increases the formation of dilute urine and is used to treat SIADH is (A) Acetazolamide (B) Amiloride (C) Desmopressin (D) Ethacrynic acid (E) Furosemide (F) Hydrochlorothiazide (G) Mannitol (H) Spironolactone (I) Triamterene (J) Tolvaptan 10. A graduate student is planning to make a high-altitude climb in South America while on vacation. He will not have time to acclimate slowly to altitude. A drug that is useful in preventing high-altitude sickness is (A) Acetazolamide (B) Amiloride (C) Demeclocycline (D) Desmopressin (E) Ethacrynic acid HISTAMINE, SERTONIN & THE ERGOT ALKALOIDS 1.Your 37-year-old patient has been diagnosed with a rare metastatic carcinoid tumor. This neoplasm is releasing serotonin, bradykinin, and several unknown peptides. The effects of serotonin in this patient are most likely to include (A) Constipation (B) Episodes of bronchospasm (C) Hypersecretion of gastric acid (D) Hypotension (E) Urinary retention 2. A 23-year-old woman suffers from recurrent episodes of angioneurotic edema with release of histamine and other mediators. Which of the following drugs is the most effective physiologic antagonist of histamine in smooth muscle? (A) Cetirizine (B) Epinephrine (C) Granisetron (D) Ranitidine (E) Sumatriptan 3. A 20-year-old woman is taking diphenhydramine for severe hay fever. Which of the following adverse effects is she most likely to report? (A) Muscarinic increase in bladder tone (B) Nausea (C) Nervousness, anxiety (D) Sedation (E) Vertigo 4. A laboratory study of new H2 blockers is planned. Which of the following will result from blockade of H2 receptors? (A) Increased cAMP (cyclic adenosine monophosphate) in cardiac muscle (B) Decreased channel opening in enteric nerves (C) Decreased cAMP in gastric mucosa (D) Increased IP3 (inositol trisphosphate) in platelets (E) Increased IP3 in smooth muscle 5. You are asked to consult on a series of cases of drug toxicities. Which of the following is a recognized adverse effect of cimetidine? (A) Blurred vision (B) Diarrhea (C) Orthostatic hypotension (D) P450 hepatic enzyme inhibition (E) Sedation 6. A 40-year-old patient is about to undergo cancer chemotherapy with a highly emetogenic (nausea- and vomiting-causing) drug combination. The antiemetic drug most likely to be included in her regimen is (A) Bromocriptine (B) Cetirizine (C) Cimetidine (D) Ketanserin (E) Ondansetron 7. The hospital Pharmacy Committee is preparing a formulary for staff use. Which of the following is a correct application of the drug mentioned? (A) Alosetron: for obstetric bleeding (B) Cetirizine: for hay fever (C) Ergonovine: for Alzheimer’s disease (D) Ondansetron: for acute migraine headache (E) Ranitidine: for Parkinson’s disease 8. A 26-year-old woman presents with amenorrhea and galactorrhea. Her prolactin level is grossly elevated (200 ng/mL vs normal 20 ng/mL). Which of the following is most useful in the treatment of hyperprolactinemia? (A) Bromocriptine (B) Cimetidine (C) Ergotamine (D) Ketanserin (E) LSD (F) Ondansetron (G) Sumatriptan 9. A 28-year-old office worker suffers from intense migraine headaches. Which of the following is a serotonin agonist useful for aborting an acute migraine headache? (A) Bromocriptine (B) Cimetidine (C) Ephedrine (D) Ketanserin (E) Loratadine (F) Ondansetron (G) Sumatriptan 10. A 33-year-old woman attempted to induce an abortion using ergotamine. She is admitted to the emergency department with severe pain in both legs. On examination, her legs are cold and pale with absent arterial pulses. Which of the following is the most useful antidote for reversing severe ergotinduced vasospasm? (A) Bromocriptine (B) Cimetidine (C) Ergotamine (D) Ketanserin (E) LSD (F) Nitroprusside (G) Sumatriptan (H) Ondansetron VASOACTIVE PEPTIDES 1. Field workers exposed to a plant toxin develop painful fluidfilled blisters. Analysis of the blister fluid reveals high concentrations of a peptide. Which of the following is a peptide that causes increased capillary permeability and edema? (A) Angiotensin II (B) Bradykinin (C) Captopril (D) Histamine (E) Losartan 2. In a laboratory study of several peptides, one is found that decreases peripheral resistance but constricts veins. Which of the following causes arteriolar vasodilation and venoconstriction? (A) Angiotensin II (B) Bradykinin (C) Endothelin-1 (D) Substance P (E) Vasoactive intestinal peptide 3. Which of the following endogenous molecules is elevated in heart failure and when given as a drug is a vasodilator with significant renal toxicity? (A) Angiotensin I (B) Angiotensin II (C) Histamine (D) Nesiritide (E) Vasoactive intestinal peptide 4. A 45-year-old painter presents with respiratory symptoms and careful workup reveals idiopathic pulmonary hypertension. Which of the following binds endothelin receptors and is approved for use in pulmonary hypertension? (A) Aliskiren I (B) Bosentan (C) Capsaicin (D) Losartan (E) Nesiritide 5. A 60-year-old financial consultant presents with severe pain in a neuronal dermatome region of her chest. This area was previously affected by a herpes zoster rash. Which of the following might be of benefit in controlling this post-herpetic pain? (A) Aliskiren (B) Aprepitant (C) Bosentan (D) Capsaicin (E) Captopril (F) Losartan (G) Nesiritide 6. In a phase 2 clinical trial in hypertensive patients, an endogenous octapeptide vasoconstrictor was found to increase in the blood of patients treated with large doses of diuretics. Which of the following is the most likely endogenous peptide? (A) Angiotensin I (B) Angiotensin II (C) Atrial natriuretic peptide (D) Bradykinin (E) Calcitonin gene-related peptide (F) Endothelin (G) Neuropeptide Y (H) Renin (I) Substance P (J) Vasoactive intestinal peptide 7. Which of the following is a vasodilator that increases in the blood or tissues of patients treated with captopril? (A) Angiotensin II (B) Bradykinin (C) Brain natriuretic peptide (D) Calcitonin gene-related peptide (E) Endothelin (F) Neuropeptide Y (G) Renin 8. Which of the following is an antagonist at NK1 receptors and is used to prevent or reduce chemotherapy-induced nausea and vomiting? (A) Angiotensin I (B) Aprepitant (C) Bosentan (D) Bradykinin (E) Brain natriuretic peptide (F) Enalapril (G) Ondansetron Prostaglandins & Other Eicosanoid 1. A 50-year-old woman with moderately severe arthritis has been treated with nonsteroidal anti-inflammatory drugs for 6 mo. She now complains of heartburn and indigestion. You give her a prescription for a drug to be taken along with the anti-inflammatory agent, but 2 d later she calls the office complaining that your last prescription has caused severe diarrhea. Which of the following is most likely to be associated with increased gastrointestinal motility and diarrhea? (A) Aspirin (B) Famotidine (C) Leukotriene LTB4 (D) Misoprostol (E) Zileuton 2. Which of the following drugs inhibits thromboxane synthesis much more effectively than prostacyclin synthesis? (A) Aspirin (B) Hydrocortisone (C) Ibuprofen (D) Indomethacin (E) Zileuton 3. A 57-year-old man has severe pulmonary hypertension and right ventricular hypertrophy. Which of the following agents causes vasodilation and may be useful in pulmonary hypertension? (A) Angiotensin II (B) Ergotamine (C) Prostaglandin PGF2 (D) Prostacyclin (E) Thromboxane 4. A 19-year-old woman complains of severe dysmenorrhea. A uterine stimulant derived from membrane lipid in the endometrium is (A) Angiotensin II (B) Oxytocin (C) Prostacyclin (PGI2) (D) Prostaglandin PGF2 (E) Serotonin 5. Inflammation is a complex tissue reaction that includes the release of cytokines, leukotrienes, prostaglandins, and peptides. Prostaglandins involved in inflammatory processes are typically produced from arachidonic acid by which of the following enzymes? (A) Cyclooxygenase-1 (B) Cyclooxygenase-2 (C) Glutathione-S-transferase (D) Lipoxygenase (E) Phospholipase A2 6. A newborn infant is diagnosed with transposition of the great vessels, wherein the aorta exits from the right ventricle and the pulmonary artery from the left ventricle. Which of the following drugs is likely to be used in preparation for surgical correction of this anomaly? (A) Aspirin (B) Leukotriene LTC4 (C) Prednisone (D) Prostaglandin PGE1 (E) Prostaglandin PGF2 7. A patient with a bleeding tendency presents in the hematology clinic. He is apparently taking large amounts of an unidentified drug that inhibits platelet activity. Which of the following is taken orally and directly and reversibly inhibits platelet cyclooxygenase? (A) Alprostadil (B) Aspirin (C) Ibuprofen (D) Leukotriene LTC4 (E) Misoprostol (F) Prednisone (G) Prostacyclin (H) Zafirlukast (I) Zileuton 8. Which of the following is a component of slow-reacting substance of anaphylaxis (SRS-A)? (A) Alprostadil (B) Aspirin (C) Leukotriene LTB4 (D) Leukotriene LTC4 (E) Misoprostol (F) Prednisone (G) Prostacyclin (H) Zafirlukast (I) Zileuton 9. A 17-year-old patient complains that he develops wheezing and severe shortness of breath whenever he takes aspirin for headache. Increased levels of which of the following may be responsible, in part, for some cases of aspirin hypersensitivity? (A) Alprostadil (B) Hydrocortisone (C) Ibuprofen (D) Leukotriene LTC4 (E) Misoprostol (F) PGE2 (G) Prostacyclin (H) Thromboxane (I) Zileuton 10. Which of the following is a leukotriene receptor blocker? (A) Alprostadil (B) Aspirin (C) Ibuprofen (D) Leukotriene LTC4 (E) Montelukast (F) Prednisone (G) Prostacyclin (H) Zileuton Nitric Oxide, Donors, & Inhibitors 1. Which one of the following is not a nitric oxide donor but causes it to be synthesized and released from endogenous precursors, resulting in vasodilation? (A) Acetylcholine (B) Arginine (C) Isosorbide mononitrate (D) Nitroglycerin (E) Nitroprusside 2. A molecule that releases nitric oxide in the blood is (A) Citrulline (B) Histamine (C) Isoproterenol (D) Nitroglycerin (E) Nitroprusside 3. The inducible isoform of nitric oxide synthase (iNOS, isoform 2) is found primarily in which of the following? (A) Cartilage (B) Eosinophils (C) Macrophages (D) Platelets (E) Vascular endothelial cells 4. The primary endogenous substrate for the enzyme nitric oxide synthase (NOS) is (A) Acetylcholine (B) Angiotensinogen (C) Arginine (D) Citrulline (E) Heme 5. Which of the following is a recognized effect of nitric oxide (NO)? (A) Arrhythmia (B) Bronchoconstriction (C) Constipation (D) Inhibition of acute graft rejection (E) Pulmonary vasodilation 6. Which of the following is an endogenous inhibitor/inactivator of nitric oxide? (A) Arginine (B) Angiotensinogen (C) Arachidonic acid (D) Hemoglobin (E) Thromboxane Drugs Used in Asthma & Chronic Obstructive Pulmonary Disease 1. One effect that theophylline, nitroglycerin, isoproterenol, and histamine have in common is (A) Direct stimulation of cardiac contractile force (B) Tachycardia (C) Bronchodilation (D) Postural hypotension (E) Throbbing headache 2. A 23-year-old woman is using an albuterol inhaler for frequent acute episodes of asthma and complains of symptoms that she ascribes to the albuterol. Which of the following is not a recognized action of albuterol? (A) Diuretic effect (B) Positive inotropic effect (C) Skeletal muscle tremor (D) Smooth muscle relaxation (E) Tachycardia 3. A 10-year-old child has severe asthma and was hospitalized 5 times between the ages of 7 and 9. He is now receiving outpatient medications that have greatly reduced the frequency of severe attacks. Which of the following is most likely to have adverse effects when used daily over long periods for severe asthma? (A) Albuterol by aerosol (B) Beclomethasone by aerosol (C) Ipratropium by inhaler (D) Prednisone by mouth (E) Theophylline in long-acting oral form 4–5. A 16-year-old patient is in the emergency department receiving nasal oxygen. She has a heart rate of 125 bpm, a respiratory rate of 40 breaths/min, and a peak expiratory flow <50% of the predicted value. Wheezing and rales are audible without a stethoscope. 4. Which of the following drugs does not have a direct bronchodilator effect? (A) Epinephrine (B) Terbutaline (C) Prednisone (D) Theophylline (E) Ipratropium 5. After successful treatment of the acute attack, the patient was referred to the outpatient clinic for follow-up treatment for asthma. Which of the following is not an established prophylactic strategy for asthma? (A) Avoidance of antigen exposure (B) Blockade of histamine receptors (C) Blockade of leukotriene receptors (D) IgE antibody blockade (E) Inhibition of phospholipase A2 6. Mr Green is a 60-year-old former smoker with cardiac disease and severe chronic obstructive pulmonary disease (COPD) associated with frequent episodes of bronchospasm. Which of the following is a bronchodilator useful in COPD and least likely to cause cardiac arrhythmia? (A) Aminophylline (B) Cromolyn (C) Epinephrine (D) Ipratropium (E) Metaproterenol (F) Metoprolol (G) Prednisone (H) Salmeterol (I) Zafirlukast (J) Zileuton 7. A 22-year-old man is brought to the emergency department after suffering seizures resulting from an overdose of a drug he has been taking. His friends state that he took the drug orally and sometimes had insomnia after taking it. Which of the following is a direct bronchodilator that is most often used in asthma by the oral route and is capable of causing insomnia and seizures? (A) Cromolyn (B) Epinephrine (C) Ipratropium (D) Metaproterenol (E) Metoprolol (F) Prednisone (G) Salmeterol (H) Theophylline (I) Zileuton 8. Which of the following in its parenteral form is life-saving in severe status asthmaticus and acts, at least in part, by inhibiting phospholipase A2? (A) Aminophylline (B) Cromolyn (C) Epinephrine (D) Ipratropium (E) Metaproterenol (F) Metoprolol (G) Prednisone (H) Salmeterol (I) Zafirlukast (J) Zileuton 9. Which of the following has a slow onset but long duration of action and is always used in combination with a corticosteroid by inhalation? (A) Aminophylline (B) Cromolyn (C) Epinephrine (D) Ipratropium (E) Metaproterenol (F) Metoprolol (G) Prednisone/prednisolone (H) Salmeterol (I) Zafirlukast (J) Zileuton 10. Oral medications are popular for the treatment of asthma in children because young children may have difficulty with the proper use of aerosol inhalers. Which of the following is an orally active inhibitor of leukotriene receptors? (A) Albuterol (B) Aminophylline (C) Ipratropium (D) Montelukast (E) Zileuton Agents Used in Cytopenias; Hematopoietic Growth Factors Questions 1–4. A 23-year-old pregnant woman is referred by her obstetrician for evaluation of anemia. She is in her fourth month of pregnancy and has no history of anemia; her grandfather had pernicious anemia. Her hemoglobin is 10 g/dL (normal, 12–16 g/dL). 1. If this woman has macrocytic anemia, an increased serum concentration of transferrin, and a normal serum concentration of vitamin B12, the most likely cause of her anemia is deficiency of which of the following? (A) Cobalamin (B) Erythropoietin (C) Folic acid (D) Intrinsic factor (E) Iron 2. The laboratory data for your pregnant patient indicate that she does not have macrocytic anemia but rather microcytic anemia. Optimal treatment of normocytic or mild microcytic anemia associated with pregnancy uses which of the following? (A) A high-fiber diet (B) Erythropoietin injections (C) Ferrous sulfate tablets (D) Folic acid supplements (E) Hydroxocobalamin injections 3. If this patient has a young child at home and is taking ironcontaining prenatal supplements, she should be warned that they are a common source of accidental poisoning in young children and advised to make a special effort to keep these pills out of her child’s reach. Toxicity associated with acute iron poisoning usually includes which of the following? (A) Dizziness, hypertension, and cerebral hemorrhage (B) Hyperthermia, delirium, and coma (C) Hypotension, cardiac arrhythmias, and seizures (D) Necrotizing gastroenteritis, shock, and metabolic acidosis (E) Severe hepatic injury, encephalitis, and coma 4. The child in the previous question did ingest the iron-containing supplements. What immediate treatment is necessary? Correction of acid-base and electrolyte abnormalities and (A) Activated charcoal (B) Oral deferasirox (C) Parenteral deferoxamine (D) Parenteral dantrolene 5. A 45-year-old male stomach cancer patient underwent tumor removal surgery. After surgery, he developed megaloblastic anemia. His anemia is caused by a deficiency of X and can be treated with Y. (A) X intrinsic factor; Y folic acid. (B) X intrinsic factor; Y vitamin B12 (C) X extrinsic factor; Y parenteral iron (D) X extrinsic factor; Y sargramostim 6. Which of the following is most likely to be required by a 5-year-old boy with chronic renal insufficiency? (A) Cyanocobalamin (B) Deferoxamine (C) Erythropoietin (D) Filgrastim (G-CSF) (E) Oprelvekin (IL-11) 7. In a patient who requires filgrastim (G-CSF) after being treated with anticancer drugs, the therapeutic objective is to prevent which of the following? (A) Allergic reactions (B) Cancer recurrence (C) Excessive bleeding (D) Hypoxia (E) Systemic infection 8. The megaloblastic anemia that results from vitamin B12 deficiency is due to inadequate supplies of which of the following? (A) Cobalamin (B) dTMP (C) Folic acid (D) Homocysteine (E) N 5-methyltetrahydrofolate Questions 9 and 10. After undergoing surgery for breast cancer, a 53-year-old woman is scheduled to receive 4 cycles of cancer chemotherapy. The cycles are to be administered every 3–5 wk. Her first cycle was complicated by severe chemotherapy-induced thrombocytopenia. 9. During the second cycle of chemotherapy, it would be appropriate to consider treating this patient with which of the following? (A) Darbepoetin alpha (B) Filgrastim (G-CSF) (C) Iron dextran (D) Oprelvekin (IL-11) (E) Vitamin B12 10. Twenty months after finishing her chemotherapy, the woman had a relapse of breast cancer. The cancer was now unresponsive to standard doses of chemotherapy. The decision was made to treat the patient with high-dose chemotherapy followed by autologous stem cell transplantation. Which of the following drugs is most likely to be used to mobilize the peripheral blood stem cells needed for the patient’s autologous stem cell transplantation? (A) Erythropoietin (B) Filgrastim (G-CSF) (C) Folic acid (D) Intrinsic factor (E) Oprelvekin (interleukin-11) Drugs Used in Coagulation Disorders Questions 1–3. A 55-year-old lawyer is brought to the emergency department 2 h after the onset of severe chest pain during a stressful meeting. He has a history of poorly controlled mild hypertension and elevated blood cholesterol but does not smoke. ECG changes (ST elevation) and cardiac enzymes confirm the diagnosis of myocardial infarction. The decision is made to attempt to open his occluded artery. 1. Which of the following drugs accelerates the conversion of plasminogen to plasmin? (A) Aminocaproic acid (B) Heparin (C) Argatroban (D) Reteplase (E) Warfarin 2. If a fibrinolytic drug is used for treatment of this man’s acute myocardial infarction, which of the following adverse drug effects is most likely to occur? (A) Acute renal failure (B) Development of antiplatelet antibodies (C) Encephalitis secondary to liver dysfunction (D) Hemorrhagic stroke (E) Neutropenia 3. If this patient undergoes a percutaneous coronary angiography procedure and placement of a stent in a coronary blood vessel, he will need to be on dual antiplatelet therapy. eg, aspirin and clopidogrel for at least a year. Which of the following most accurately describes the mechanism of action of clopidogrel? (A) Clopidogrel directly binds to the platelet ADP receptors (B) Clopidogrel irreversibly inhibits cyclooxygenase (C) Clopidogrel facilitates the action of antithrombin III (D) The active metabolite of clopidogrel binds to the platelet ADP receptors (E) The active metabolite of clopidogrel binds to the platelet glycoprotein IIb/IIIa receptors 4. The above graph shows the plasma concentration of free warfarin as a function of time for a patient who was treated with 2 other agents, drugs B and C, on a daily basis at constant dosage starting at the times shown. Which of the following is the most likely explanation for the observed changes in warfarin concentration? (A) Drug B displaces warfarin from plasma proteins; drug C displaces warfarin from tissue-binding sites (B) Drug B inhibits hepatic metabolism of warfarin; drug C displaces drug B from tissue-binding sites (C) Drug B stimulates hepatic metabolism of warfarin; drug C displaces warfarin from plasma protein (D) Drug B increases renal clearance of warfarin; drug C inhibits hepatic metabolism of drug B Questions 5–7. A 58-year-old woman with chronic hypertension and diabetes mellitus was recently admitted to the hospital for congestive heart failure and new onset atrial fibrillation. She is now seeing you after discharge and, though feeling better, is still in atrial fibrillation. An echocardiogram shows an ejection fraction of 40%; there are no valvular abnormalities. An ECG reveals only atrial fibrillation. You calculate her risk using the CHADS(2) system and the score indicates that she requires anticoagulation rather than antiplatelet therapy. 5. You are discussing the risks and benefits of anticoagulation therapy with her, including the option of using direct thrombin inhibitors. Which of the following anticoagulants is a direct inhibitor of thrombin? (A) Abciximab (B) Dabigatran (C) Rivaroxaban (D) Warfarin 6. She tells you that her main reason for not wanting oral anticoagulation is that she does not want to come to clinic for frequent blood draws. You agree on an oral alternative and start her on apixaban. You counsel her extensively on the importance of taking the medication each day, as suddenly stopping can lead to (A) Anaphylaxis (B) Excess bleeding (C) Increase in INR (D) Stroke (E) Thrombocytopenia 7. She is excited about not having to come in for blood tests but wonders if there is a test, just in case the doctors need to know. Which of the following tests would provide accurate information about the coagulation status of a patient taking apixaban? (A) aPTT (B) Factor X test (C) INR (D) PT test Questions 8 and 9. A 67-year-old woman presents with pain in her left thigh muscle. Duplex ultrasonography indicates the presence of deep vein thrombosis (DVT) in the affected limb. 8. The decision was made to treat this woman with enoxaparin. Relative to unfractionated heparin, enoxaparin (A) Can be used without monitoring the patient’s aPTT (B) Has a shorter duration of action (C) Is less likely to have a teratogenic effect (D) Is more likely to be given intravenously (E) Is more likely to cause thrombosis and thrombocytopenia 9. During the next week, the patient was started on warfarin and her enoxaparin was discontinued. Two months later, she returned after a severe nosebleed. Laboratory analysis revealed an INR (international normalized ratio) of 7.0 (INR value in such a warfarin-treated patient should be 2.0–3.0). To prevent severe hemorrhage, the warfarin should be discontinued and this patient should be treated immediately with which of the following? (A) Aminocaproic acid (B) Desmopressin (C) Factor VIII (D) Protamine (E) Vitamin K1 10. A patient develops severe thrombocytopenia in response to treatment with unfractionated heparin and still requires parenteral anticoagulation. The patient is most likely to be treated with which of the following? (A) Abciximab (B) Bivalirudin (C) Tirofiban (D) Plasminogen (E) Vitamin K1 Agents Used in Dyslipidemia 1. PJ is a 4.5-year-old boy. At his checkup, the pediatrician notices cutaneous xanthomas and orders a lipid panel. Repeated measures confirm that the patient’s serum cholesterol levels are high (936 mg/dL). Further testing confirms a diagnosis of homozygous familial hypercholesterolemia. Which of the following interventions will be least effective in this patient? (A) Atorvastatin (B) Ezetimibe (C) Lomitapide (D) Mipomersen (E) Niacin 2. A 46-year-old woman with a history of hyperlipidemia was treated with a drug. The chart below shows the results of the patient’s fasting lipid panel before treatment and 6 mo after initiating drug therapy. Normal values are also shown. Which of the following drugs is most likely to be the one that this patient received? (A) Colestipol (B) Ezetimibe (C) Gemfibrozil (D) Lovastatin (E) Niacin Questions 3–6. A 35-year-old woman appears to have familial combined hyperlipidemia. Her serum concentrations of total cholesterol, LDL cholesterol, and triglyceride are elevated. Her serum concentration of HDL cholesterol is somewhat reduced. 3. Which of the following drugs is most likely to increase this patient’s triglyceride and VLDL cholesterol concentrations when used as monotherapy? (A) Atorvastatin (B) Cholestyramine (C) Ezetimibe (D) Gemfibrozil (E) Niacin 4. If this patient is pregnant, which of the following drugs should be avoided because of a risk of harming the fetus? (A) Cholestyramine (B) Ezetimibe (C) Fenofibrate (D) Niacin (E) Pravastatin 5. The patient is started on gemfibrozil. Which of the following is a major mechanism of gemfibrozil’s action? (A) Increased excretion of bile acid salts (B) Increased expression of high-affinity LDL receptors (C) Increased secretion of VLDL by the liver (D) Increased triglyceride hydrolysis by lipoprotein lipase (E) Reduced uptake of dietary cholesterol 6. Which of the following is a major toxicity associated with gemfibrozil therapy? (A) Bloating and constipation (B) Cholelithiasis (C) Hyperuricemia (D) Liver damage (E) Severe cardiac arrhythmia Questions 7–10. A 43-year-old man has heterozygous familial hypercholesterolemia. His serum concentrations of total cholesterol and LDL are markedly elevated. His serum concentration of HDL cholesterol, VLDL cholesterol, and triglycerides are normal or slightly elevated. The patient’s mother and older brother died of myocardial infarctions before the age of 50. This patient recently experienced mild chest pain when walking upstairs and has been diagnosed as having angina of effort. The patient is somewhat overweight. He drinks alcohol most evenings and smokes about 1 pack of cigarettes per week. 7. Consumption of alcohol is associated with which of the following changes in serum lipid concentrations? (A) Decreased chylomicrons (B) Decreased HDL cholesterol (C) Decreased VLDL cholesterol (D) Increased LDL cholesterol (E) Increased triglyceride If the patient has a history of gout, which of the following drugs is most likely to exacerbate this condition? (A) Colestipol (B) Ezetimibe (C) Gemfibrozil (D) Niacin (E) Simvastatin 9. After being counseled about lifestyle and dietary changes, the patient was started on atorvastatin. During his treatment with atorvastatin, it is important to routinely monitor serum concentrations of which of the following? (A) Blood urea nitrogen (B) Alanine and aspartate aminotransferase (C) Platelets (D) Red blood cells (E) Uric acid 10. Six months after beginning atorvastatin, the patient’s total and LDL cholesterol concentrations remained above normal, and he continued to have anginal attacks despite good adherence to his antianginal medications. His physician decided to add ezetimibe. Which of the following is the most accurate description of ezetimibe’s mechanism of an action? (A) Decreased lipid synthesis in adipose tissue (B) Decreased secretion of VLDL by the liver (C) Decreased gastrointestinal absorption of cholesterol (D) Increased endocytosis of HDL by the liver (E) Increased lipid hydrolysis by lipoprotein lipase. NSAIDs, Acetaminophen,& Drugs Used in Rheumatoid Arthritis& Gout 1. Among NSAIDs, aspirin is unique because it (A) Irreversibly inhibits its target enzyme (B) Prevents episodes of gouty arthritis with long-term use (C) Reduces fever (D) Reduces the risk of colon cancer (E) Selectively inhibits the COX-2 enzyme 2. Which of the following is an analgesic and antipyretic drug that lacks an anti-inflammatory action? (A) Acetaminophen (B) Celecoxib (C) Colchicine (D) Indomethacin (E) Probenecid 3. A 16-year-old girl comes to the emergency department suffering from the effects of an aspirin overdose. Which of the following syndromes is this patient most likely to exhibit as a result of this drug overdose? (A) Bone marrow suppression and possibly aplastic anemia (B) Fever, hepatic dysfunction, and encephalopathy (C) Hyperthermia, metabolic acidosis, and coma (D) Rapid, fulminant hepatic failure (E) Rash, interstitial nephritis, and acute renal failure 4. Which of the following drugs is most likely to increase serum concentrations of conventional doses of methotrexate, a weak acid that is primarily cleared in the urine? (A) Acetaminophen (B) Allopurinol (C) Colchicine (D) Hydroxychloroquine (E) Probenecid 5. The main advantage of ketorolac over aspirin is that ketorolac (A) Can be combined more safely with an opioid such as codeine (B) Can be obtained as an over-the-counter agent (C) Does not prolong the bleeding time (D) Is available in a parenteral formulation that can be injected intramuscularly or intravenously (E) Is less likely to cause acute renal failure in patients with some preexisting degree of renal impairment 6. An 18-month-old boy dies from an accidental overdose of acetaminophen. Which of the following is the most likely cause of this patient’s death? (A) Arrhythmia (B) Hemorrhagic stroke (C) Liver failure (D) Noncardiogenic pulmonary edema (E) Ventilatory failure Questions 7 and 8. A 52-year-old woman presented with intense pain, warmth, and redness in the first toe on her left foot. Examination of fluid withdrawn from the inflamed joint revealed crystals of uric acid. 7. In the treatment of this woman’s acute attack of gout, a high dose of colchicine will reduce the pain and inflammation. However, many physicians prefer to treat acute gout with a corticosteroid or indomethacin because high doses of colchicine are likely to cause (A) Behavioral changes that include psychosis (B) High blood pressure (C) Rash (D) Severe diarrhea (E) Sudden gastrointestinal bleeding 8. Over the next 7 mo, the patient had 2 more attacks of acute gout. Her serum concentration of uric acid was elevated. The decision was made to put her on chronic drug therapy to try to prevent subsequent attacks. Which of the following drugs could be used to decrease this woman’s rate of production of uric acid? (A) Allopurinol (B) Aspirin (C) Colchicine (D) Hydroxychloroquine (E) Probenecid Questions 9 and 10. A 54-year-old woman presented with signs and symptoms consistent with an early stage of rheumatoid arthritis. The decision was made to initiate NSAID therapy. 9. Which of the following patient characteristics is the most compelling reason for avoiding celecoxib in the treatment of her arthritis? (A) History of alcohol abuse (B) History of gout (C) History of myocardial infarction (D) History of osteoporosis (E) History of peptic ulcer disease 10. Although the patient’s disease was adequately controlled with an NSAID and methotrexate for some time, her symptoms began to worsen and radiologic studies of her hands indicated progressive destruction in the joints of several fingers. Treatment with another second-line agent for rheumatoid arthritis was considered. Which of the following is a parenterally administered DMARD whose mechanism of antiinflammatory action is antagonism of tumor necrosis factor? (A) Cyclosporine (B) Etanercept (C) Penicillamine (D) Phenylbutazone (E) Sulfasalazine CHEMOTHERAPEUTIC DRUGS 1. A 4-year-old child is brought to the hospital after ingesting pills that a parent had used for bacterial dysentery when traveling outside the United States. The child has been vomiting for more than 24 h and has had diarrhea with green stools. She is now lethargic with an ashen color. Other signs and symptoms include hypothermia, hypotension, and abdominal distention. The drug most likely to be the cause of this problem is (A) Ampicillin (B) Azithromycin (C) Chloramphenicol (D) Doxycycline (E) Erythromycin 2. The mechanism of antibacterial action of tetracycline involves (A) Antagonism of bacterial translocase activity (B) Binding to a component of the 50S ribosomal subunit (C) Inhibition of DNA-dependent RNA polymerase (D) Interference with binding of aminoacyl-tRNA to bacterial ribosomes (E) Selective inhibition of ribosomal peptidyl transferases 3. Clarithromycin and erythromycin have very similar spectra of antimicrobial activity. The major advantage of clarithromycin is that it (A) Does not inhibit hepatic drug-metabolizing enzymes (B) Eradicates mycoplasmal infections in a single dose (C) Has greater activity against H pylori (D) Is active against methicillin-resistant strains of staphylococci (E) Is active against strains of streptococci that are resistant to erythromycin 4. The primary mechanism of resistance of gram-positive organisms to macrolide antibiotics including erythromycin is (A) Changes in the 30S ribosomal subunit (B) Decreased drug permeability of the cytoplasmic membrane (C) Formation of drug-inactivating acetyltransferases (D) Formation of esterases that hydrolyze the lactone ring (E) Methylation of binding sites on the 50S ribosomal Subunit 5. A 26-year-old woman was treated for a suspected chlamydial infection at a neighborhood clinic. She was given a prescription for oral doxycycline to be taken for 14 d. Three weeks later, she returned to the clinic with a mucopurulent cervicitis. On questioning she admitted not having the prescription filled. The best course of action at this point would be to (A) Delay drug treatment until the infecting organism is identified (B) Rewrite the original prescription for oral doxycycline (C) Treat her in the clinic with a single oral dose of azithromycin (D) Treat her in the clinic with an intravenous dose of amoxicillin (E) Write a prescription for oral erythromycin for 10 d 6. A 55-year-old patient with a prosthetic heart valve is to undergo a periodontal procedure involving scaling and root planing. Several years ago, the patient had a severe allergic reaction to procaine penicillin G. Regarding prophylaxis against bacterial endocarditis, which one of the following drugs taken orally is most appropriate? (A) Amoxicillin 10 min before the procedure (B) Clindamycin 1 h before the procedure (C) Erythromycin 1 h before the procedure and 4 h after the procedure (D) Vancomycin 15 min before the procedure (E) No prophylaxis is needed because this patient is in the negligible risk category Questions 7–9. A 24-year-old woman comes to a clinic with complaints of dry cough, headache, fever, and malaise, which have lasted 3 or 4 d. She appears to have some respiratory difficulty, and chest examination reveals rales but no other obvious signs of pulmonary involvement. However, extensive patchy infiltrates are seen on chest x-ray film. Gram stain of expectorated sputum fails to reveal any bacterial pathogens. The patient mentions that a colleague at work had similar symptoms to those she is experiencing. The patient has no history of serious medical problems. She takes loratadine for allergies and supplementary iron tablets, and she drinks at least 6 cups of caffeinated coffee per day. The physician makes an initial diagnosis of communityacquired pneumonia. 7. Regarding the treatment of this patient, which of the following drugs is most suitable? (A) Ampicillin (B) Clindamycin (C) Doxycycline (D) Linezolid (E) Vancomycin 8. If this patient were to be treated with erythromycin, she should (A) Avoid exposure to sunlight (B) Avoid taking supplementary iron tablets (C) Decrease her intake of caffeinated beverages (D) Have her plasma urea nitrogen or creatinine checked before treatment (E) Temporarily stop taking loratadine 9. A 5-d course of treatment for community-acquired pneumonia would be effective in this patient with little risk of drug interactions if the drug prescribed were (A) Azithromycin (B) Clindamycin (C) Doxycycline (D) Erythromycin (E) Vancomycin 10. Concerning quinupristin-dalfopristin, which statement is accurate? (A) Active in treatment of infections caused by E faecalis (B) An effective drug in treatment of multidrug-resistant streptococcal infections (C) Bacteriostatic (D) Hepatotoxicity has led to FDA drug alerts (E) Increase the activity of hepatic drug-metabolizing Enzymes. 1. Regarding the mechanism of action of aminoglycosides, the drugs (A) Are bacteriostatic (B) Bind to the 50S ribosomal subunit (C) Cause misreading of the code on the mRNA template (D) Inhibit peptidyl transferase (E) Stabilize polysomes 2. A 72-kg patient with creatinine clearance of 80 mL/min has a gram-negative infection. Amikacin is administered intramuscularly at a dose of 5 mg/kg every 8 h, and the patient begins to respond. After 2 d, creatinine clearance declines to 40 mL/min. Assuming that no information is available about amikacin plasma levels, what would be the most reasonable approach to management of the patient at this point? (A) Administer 5 mg/kg every 12 h (B) Decrease the dosage to daily total of 200 mg (C) Decrease the dosage to 180 mg every 8 h (D) Discontinue amikacin and switch to gentamicin (E) Maintain the patient on the present dosage and test auditory function 3. All of the following statements about the clinical uses of the aminoglycosides are accurate except (A) Effective in the treatment of infections caused by anaerobes such as Bacteroides fragilis (B) Gentamycin is used with ampicillin for synergistic effects in the treatment of enterococcal endocarditis (C) In the treatment of a hospital-acquired infection caused by Serratia marcescens, netilmicin is more effective than streptomycin (D) Often used with cephalosporins in the empiric treatment of life-threatening bacterial infections (E) Owing to their polar nature, aminoglycosides are not absorbed after oral administration 4. Which statement is accurate regarding the antibacterial action of the aminoglycoside amikacin? (A) Antibacterial activity is often reduced by the presence of an inhibitor of cell wall synthesis (B) Antibacterial action is not concentration-dependent (C) Antibacterial action is time-dependent (D) Efficacy is directly proportional to the duration of time that the plasma level is greater than the minimal inhibitory concentration (E) The drug continues to exert antibacterial effects even after plasma levels decrease below detectable levels 5. An adult patient (weight 80 kg) has bacteremia suspected to be due to a gram-negative rod. Tobramycin is to be administered using a once-daily dosing regimen, and the loading dose must be calculated to achieve a peak plasma level of 20 mg/L. Assume that the patient has normal renal function. Pharmacokinetic parameters of tobramycin in this patient are as follows: Vd = 30 L; t1/2 = 3 h; CL = 80 mL/min. What loading dose should be given? (A) 100 mg (B) 200 mg (C) 400 mg (D) 600 mg (E) 800 mg 6. A 76-year-old man is seen in a hospital emergency department complaining of pain in and behind the right ear. Physical examination shows edema of the external otic canal with purulent exudate and weakness of the muscles on the right side of the face. The patient informs the physician that he is a diabetic. Gram stain of the exudate from the ear shows many polymorphonucleocytes and gram-negative rods, and samples are sent to the microbiology laboratory for culture and drug susceptibility testing. A preliminary diagnosis is made of external otitis. At this point, which of the following is most appropriate? (A) Amikacin should be injected intramuscularly and the patient should be sent home (B) Analgesics should be prescribed, but antibiotics should be withheld pending microbiological results (C) Oral cefaclor should be prescribed together with analgesics, and the patient should be sent home (D) The patient should be hospitalized and treatment started with imipenem-cilastatin (E) The patient should be hospitalized and treatment started with gentamicin plus ticarcillin 7. Regarding the toxicity of aminoglycosides which statement is accurate? (A) Gentamicin and tobramycin are the least likely to cause renal damage (B) Ototoxicity due to amikacin and gentamicin includes vestibular dysfunction, which is often irreversible (C) Ototoxicity is reduced if loop diuretics are used to facilitate the renal excretion of aminoglycoside antibiotics (D) Reduced blood creatinine is an early sign of aminoglycoside nephrotoxicity (E) Skin reactions are very rare following topical use of Neomycin 8. This drug has characteristics almost identical to those of gentamicin but has much weaker activity in combination with penicillin against enterococci. (A) Amikacin (B) Erythromycin (C) Netilmicin (D) Spectinomycin (E) Tobramycin 9. Your 23-year-old female patient is pregnant and has gonorrhea. The medical history includes anaphylaxis following exposure to amoxicillin. The most appropriate drug to use is (A) Azithromycin (B) Cefixime (C) Ceftriaxone (D) Ciprofloxacin (E) Doxycycline 10. Which statement about “once-daily” dosing with aminoglycosides is not accurate? (A) Dose adjustment is less important in renal dysfunction (B) It is convenient for outpatient treatment (C) Less nursing time is required for drug administration (D) Often less side effects than multiple (conventional) dosing regimens (E) Underdosing is less of a problem 1. Trimethoprim-sulfamethoxazole is established to be effective against which of the following opportunistic infections in the AIDS patient? (A) Cryptococcal meningitis (B) Herpes simplex (C) Oral candidiasis (D) Toxoplasmosis (E) Tuberculosis 2. A 65-year-old woman has returned from a vacation abroad suffering from traveler’s diarrhea, and her problem has not responded to antidiarrheal drugs. A pathogenic gram-negative bacillus is suspected. Which drug is most likely to be effective in the treatment of this patient? (A) Ampicillin (B) Ofloxacin (C) Sulfadiazine (D) Trimethoprim (E) Vancomycin 3. Which statement about the clinical use of sulfonamides is false? (A) Active against C trachomatis and can be used topically for treatment of chlamydial infections of the eye (B) Are not effective as sole agents in the treatment of prostatitis (C) Effective in Rocky Mountain spotted fever (D) In some bacterial strains resistance occurs via increased PABA formation (E) Reduced intracellular uptake is a mechanism of sulfonamide resistance in some bacterial strains 4. A 31-year-old man has gonorrhea. He has no drug allergies, but a few years ago acute hemolysis followed use of an antimalarial drug. The physician is concerned that the patient has an accompanying urethritis caused by C trachomatis, although no cultures or enzyme tests have been performed. Which of the following drugs will be reliably effective against both gonococci and C trachomatis and safe to use in this patient? (A) Cefixime (B) Ciprofloxacin (C) Spectinomycin (D) Sulfamethoxazole-trimethoprim (E) None of the above 5. Which statement about the fluoroquinolones is accurate? (A) Antacids increase their oral bioavailability (B) Contraindicated in patients with hepatic dysfunction (C) Fluoroquinolones are drugs of choice in a 6-year-old child with a urinary tract (D) Gonococcal resistance to fluoroquinolones may involve changes in DNA gyrase (E) Modification of moxifloxacin dosage is required in patients when creatinine clearance is less than 50 mL/ min 6. A 40-year-old man complains of periodic bouts of diarrhea with lower abdominal cramping and intermittent rectal bleeding. Seen in the clinic, he appears well nourished, with blood pressure in the normal range. Examination reveals moderate abdominal pain and tenderness. His current medications are limited to loperamide for his diarrhea. Sigmoidoscopy reveals mucosal edema, friability, and some pus. Laboratory findings include mild anemia and decreased serum albumin. Microbiologic examination via stool cultures and mucosal biopsies do not reveal any evidence for bacterial, amebic, or cytomegalovirus involvement. The most appropriate drug to use in this patient is (A) Ampicillin (B) Doxycycline (C) Norfloxacin (D) Sulfasalazine (E) Trimethoprim-sulfamethoxazole 7. Which adverse effect is most common with sulfonamides? (A) Fanconi’s aminoaciduria syndrome (B) Hematuria (C) Kernicterus in the newborn (D) Neurologic dysfunction (E) Skin rash 8. Which drug is effective in the treatment of nocardiosis and, in combination with pyrimethamine, is prophylactic against Pneumocystis jirovecii infections in AIDS patients? (A) Amoxicillin (B) Erythromycin (C) Levofloxacin (D) Sulfadiazine (E) Trimethoprim 9. Which statement about ciprofloxacin is accurate? (A) Antagonism occurs if used with dihydrofolate reductase inhibitors (B) Ciprofloxacin is active against MRSA strains of staphylococci (C) Most “first-time” urinary tract infections are resistant to ciprofloxacin (D) Organisms that commonly cause ear infections are highly resistant (E) Tendinitis may occur during treatment 10. Supplementary folinic acid may prevent anemia in folatedeficient persons who use this drug; it is a weak base achieving tissue levels similar to those in plasma (A) Ciprofloxacin (B) Levofloxacin (C) Linezolid (D) Sulfamethoxazole (E) Trimethoprim 1. The primary reason for the use of drug combinations in the treatment of tuberculosis is to (A) Delay or prevent the emergence of resistance (B) Ensure patient compliance with the drug regimen (C) Increase antibacterial activity synergistically (D) Provide prophylaxis against other bacterial infections (E) Reduce the incidence of adverse effects Questions 2–5. A 21-year-old woman from Southeast Asia has been staying with family members in the United States for the last 3 mo and is looking after her sister’s preschool children during the day. Because she has difficulty with the English language, her sister escorts her to the emergency department of a local hospital. She tells the staff that her sister has been feeling very tired for the last month, has a poor appetite, and has lost weight. The patient has been feeling somewhat better lately except for a cough that produces a greenish sputum, sometimes specked with blood. With the exception of rales in the left upper lobe, the physical examination is unremarkable and she does not seem to be acutely ill. Laboratory values show a white count of 12,000/μL and a hematocrit of 33%. Chest x-ray film reveals an infiltrate in the left upper lobe with a possible cavity. A Gram-stained smear of the sputum shows mixed flora with no dominance. An acid-fast stain reveals many thin rods of pinkish hue. A preliminary diagnosis is made of pulmonary tuberculosis. Sputum is sent to the laboratory for culture. 2. At this point, the most appropriate course of action is to (A) Hospitalize the patient and start treatment with 4 antitubercular drugs (B) Hospitalize the patient and start treatment with rifampin (C) Prescribe isoniazid for prophylaxis and send the patient home to await culture results (D) Provide no drugs and send the patient home to await culture results (E) Treat the patient with isoniazid plus rifampin 3. Which drug regimen should be initiated in this patient when treatment is started? (A) Amikacin, isoniazid, pyrazinamide, streptomycin (B) Ciprofloxacin, cycloserine, isoniazid, PAS (C) Ethambutol, isoniazid, pyrazinamide, rifampin (D) Isoniazid, pyrazinamide, rifampin, streptomycin (E) PAS, pyrazinamide, rifabutin, streptomycin 4. Which statement concerning the possible use of isoniazid (INH) in this patient is false? (A) Dyspnea, flushing, palpitations, and sweating may occur after ingestion of tyramine-containing foods (B) In patients from Southeast Asia, lower maintenance doses are necessary (C) Peripheral neuritis may occur during treatment (D) The patient should take pyridoxine daily (E) The risk of the patient developing hepatitis from INH is less than 2% 5. On her release from the hospital, the patient is advised not to rely solely on oral contraceptives to prevent pregnancy because they may be less effective while she is being maintained on antimycobacterial drugs. The agent most likely to interfere with the action of oral contraceptives is (A) Amikacin (B) Ethambutol (C) Isoniazid (D) Pyrazinamide (E) Rifampin 6. A patient with AIDS and a CD4 cell count of 100/μL has persistent fever and weight loss associated with invasive pulmonary disease due to M avium complex (MAC). Optimal management of this patient is to (A) Choose an antibiotic based on drug susceptibility of the cultured organism (B) Initiate a two-drug regimen of INH and pyrazinamide (C) Prescribe rifabutin because it prevents the development of MAC bacteremia (D) Start treatment with the combination of azithromycin, ethambutol, and rifabutin (E) Treat with trimethoprim-sulfamethoxazole 7. A 10-year-old boy has uncomplicated pulmonary tuberculosis. After initial hospitalization, he is now being treated at home with isoniazid, rifampin, and ethambutol. Which statement about this case is accurate? (A) A baseline test of auditory function test is essential before drug treatment is initiated (B) His mother, who takes care of him, does not need INH prophylaxis (C) His 3-year-old sibling should receive INH prophylaxis (D) Polyarthralgia is a potential adverse effect of the drugs the boy is taking (E) The potential nephrotoxicity of the prescribed drugs warrants periodic assessment of renal function 8. Which statement about antitubercular drugs is accurate? (A) Antimycobacterial actions of streptomycin involve inhibition of arabinosyltransferases (B) Cross-resistance of M tuberculosis to isoniazid and pyrazinamide is common (C) Ocular toxicity of ethambutol is prevented by thiamine (D) Pyrazinamide treatment should be discontinued immediately if hyperuricemia occurs (E) Resistance to ethambutol involves mutations in the emb gene 9. Once-weekly administration of which of the following antibiotics has prophylactic activity against bacteremia caused by M avium complex in AIDS patients? (A) Acedapsone (B) Azithromycin (C) Clarithromycin (D) Kanamycin (E) Rifabutin 10. Risk factors for multidrug-resistant tuberculosis include (A) A history of treatment of tuberculosis without rifampin (B) Recent immigration from Asia and living in an area of over 4% isoniazid resistance (C) Recent immigration from Latin America (D) Residence in regions where isoniazid resistance is known to exceed 4% (E) All of the above 1. Interactions between this drug and cell membrane components can result in the formation of pores lined by hydrophilic groups present in the drug molecule. (A) Caspofungin (B) Flucytosine (C) Griseofulvin (D) Nystatin (E) Terbinafine 2. Which statement about fluconazole is accurate? (A) Does not penetrate the blood-brain barrier (B) Drug of choice in treatment of aspergillosis (C) Induces hepatic drug-metabolizing enzymes (D) Has the least effect of all azoles on drug metabolism (E) Oral bioavailability is less than that of ketoconazole Questions 3–5. A 37-year-old woman with leukemia was undergoing chemotherapy with intravenous antineoplastic drugs. During treatment, she developed a systemic infection from an opportunistic pathogen. There was no erythema or edema at the catheter insertion site. A white vaginal discharge was observed. After appropriate specimens were obtained for culture, empiric antibiotic therapy was started with gentamicin, nafcillin, and ticarcillin intravenously. This regimen was maintained for 72 h, during which time the patient’s condition did not improve significantly. Her throat was sore, and white plaques had appeared in her pharynx. On day 4, none of the cultures had shown any bacterial growth, but both the blood and urine cultures grew out Candida albicans. 3. At this point, the best course of action is to (A) Continue current antibiotics and start griseofulvin (B) Continue current antibiotics and start amphotericin B (C) Stop current antibiotics and start itraconazole (D) Stop current antibiotics and start amphotericin B (E) Stop current antibiotics and start terbinafine 4. If amphotericin B is administered, the patient should be premedicated with (A) Diphenhydramine (B) Ibuprofen (C) Prednisone (D) Any or all of the above (E) None of the above 5. Candida is a major cause of nosocomial bloodstream infection. The opportunistic fungal infection in this patient could have been prevented by administration of (A) Caspofungin (B) Flucytosine (C) Nystatin (D) Voriconazole (E) None of the above Questions 6–7. A 28-year-old man living on the East Coast was transferred by his employer to California for several months. On his return, he complains of having influenza-like symptoms with fever and a cough. He also has red, tender nodules on his shins. His physician suspects that these symptoms are due to coccidioidomycosis contracted during his stay in California. 6. This patient should be treated immediately with (A) Amphotericin B (B) Caspofungin (C) Ketoconazole (D) Terbinafine (E) None of these drugs 7. Which is the drug of choice if this patient is suffering from persistent lung lesions or disseminated disease caused by Coccidioides immitis? (A) Amphotericin B (B) Flucytosine (C) Itraconazole (D) Micofungin (E) Terbinafine 8. Which drug is least likely to be effective in the treatment of esophageal candidiasis if it is used by the oral route? (A) Clotrimazole (B) Griseofulvin (C) Ketoconazole (D) Itraconazole (E) Nystatin 9. Serious cardiac effects have occurred when this drug was taken by patients using the antihistamines astemizole or terfenadine (A) Amphotericin B (B) Griseofulvin (C) Ketoconazole (D) Terbinafine (E) Voriconazole 10. Regarding the clinical use of liposomal formulations of amphotericin B, which statement is accurate? (A) Amphotericin B affinity for these lipids is greater than affinity for ergosterol (B) Less expensive to use than conventional amphotericin B (C) More effective in fungal infections because they increase tissue uptake of amphotericin B (D) They decrease the nephrotoxicity of amphotericin B (E) They have a wider spectrum of antifungal activity than conventional formulations of amphotericin B 1. Which statement about the mechanisms of action of antiviral drugs is accurate? (A) Acyclovir has no requirement for activation by phosphorylation (B) Ganciclovir inhibits viral DNA polymerase but does not cause chain termination (C) Increased activity of host cell ribonucleases that degrade viral mRNA is one of the actions of interferon- (D) The initial step in activation of foscarnet in HSVinfected cells is its phosphorylation by thymidine kinase (E) The reverse transcriptase of HIV is 30–50 times more sensitive to inhibition by fosamprenavir than host cell DNA polymerases Questions 2 and 3. A 30-year-old male patient who is HIVpositive and symptomatic has a CD4 count of 250/μL and a viral RNA load of 15,000 copies/mL. His treatment involves a 3-drug antiviral regimen consisting of zidovudine, didanosine, and ritonavir. The patient is taking acyclovir for a herpes infection and ketoconazole for oral candidiasis. He now complains of anorexia, nausea and vomiting, and abdominal pain. His abdomen is tender in the epigastric area. Laboratory results reveal an amylase activity of 220 U/L, and a preliminary diagnosis is made of acute pancreatitis. 2. If this patient has acute pancreatitis, the drug most likely to be responsible is (A) Acyclovir (B) Didanosine (C) Ketoconazole (D) Ritonavir (E) Zidovudine 3. In the further treatment of this patient, the drug causing the pancreatitis should be withdrawn and replaced by (A) Atazanavir (B) Cidofovir (C) Foscarnet (D) Lamivudine (E) Ribavirin 4. In an accidental needlestick, an unknown quantity of blood from an AIDS patient is injected into a resident physician. The most recent laboratory report on the AIDS patient shows a CD4 count of 20/μL and a viral RNA load of greater than 107 copies/mL. The most appropriate course of action regarding treatment of the resident is to (A) Determine whether HIV transmission has occurred by monitoring the patient’s blood (B) Treat with a single high dose of zidovudine (C) Treat with full doses of zidovudine for 4 wk (D) Treat with single doses of zidovudine and indinavir (E) Treat with zidovudine plus lamivudine plus ritonavir for 4 wk Questions 5 and 6. A patient with AIDS has a CD4 count of 45/L. He is being maintained on a 3-drug regimen of indinavir, didanosine, and zidovudine. For prophylaxis against opportunistic infections, he is also receiving cidofovir, fluconazole, rifabutin, and trimethoprim-sulfamethoxazole. 5. The drug most likely to suppress herpetic infections and provide prophylaxis against CMV retinitis in this patient is (A) Fluconazole (B) Cidofovir (C) Indinavir (D) Rifabutin (E) Trimethoprim-sulfamethoxazole 6. The dose of indinavir in this patient may need to be increased above normal. This is because (A) Fluconazole slows gastric emptying (B) Ganciclovir increases the renal clearance of indinavir (C) Gastric absorption is inhibited by fluconazole (D) Rifabutin increases hepatic drug metabolism (E) Sulfamethoxazole increases indinavir plasma protein binding 7. A 27-year-old nursing mother is diagnosed as suffering from genital herpes. She has a history of this viral infection. Previously, she responded to a drug used topically. Apart from her current problem, she is in good health. Which drug to be used orally is most likely to be prescribed at this time? (A) Amantadine (B) Foscarnet (C) Ritonavir (D) Trifluridine (E) Valacyclovir 8. Oral formulations of this drug should not be used in a pregnant AIDS patient because they contain propylene glycol. One of the characteristic adverse effects of the drug is hyperpigmentation on the palms of the hands and soles of the feet, especially in African-American patients. (A) Amprenavir (B) Emtricitabine (C) Efavirenz (D) Fosamprenavir (E) Zalcitabine 9. Which of the following statements about interferon-is false? (A) At the start of treatment, most patients experience flulike symptoms (B) Indications include treatment of genital warts (C) It is used in the management of hepatitis B and C (D) Lamivudine interferes with its activity against hepatitis B (E) Toxicity includes bone marrow suppression 10. More than 90% of this drug is excreted in the urine in intact form. Because its urinary solubility is low, patients should be well hydrated to prevent nephrotoxicity. Which drug is described? (A) Acyclovir (B) Efavirenz (C) Indinavir (D) Trifluridine (E) Zidovudine 1. Infections caused by gram-negative bacilli have occurred when this cationic surfactant has been used as a skin antiseptic. (A) Acetic acid (B) Benzalkonium chloride (C) Lindane (D) Hexachlorophene (E) Thimerosal Questions 2 and 3. A young woman is brought to a hospital emergency department with intense abdominal pain of 2 d duration. The pain has spread to the right lower quadrant and is accompanied by nausea, vomiting, and fever. She arrives at the emergency department with a blood pressure of 85/45, pulse 120/min, and temperature 40C. Her abdomen has a board-like rigidity with diffuse pain to palpation. Laboratory values include the following: WBC 20,000/L and creatinine 1.5 mg/dL. After abdominal x-ray films are taken, a preliminary diagnosis of abdominal sepsis is made, possibly resulting from bowel perforation. After appropriate samples are sent to the laboratory for culture, the patient is hospitalized, and antimicrobial therapy is started with intravenous ampicillin and gentamicin. 2. Regarding the treatment of this patient, which statement is accurate? (A) A drug active against anaerobes should be included in the antimicrobial drug regimen (B) Cultures are pointless because this is probably a mixed infection (C) Empiric antibiotic therapy of abdominal sepsis should always include a third-generation cephalosporin (D) Gram stain of the blood would provide positive identification of the specific organism involved in this infection (E) The combination of ampicillin and gentamicin provides good coverage for all likely pathogens 3. If the antibiotic regimen in this patient is modified to include metronidazole (A) Ampicillin should be excluded from the regimen (B) Coverage will be extended to methicillin-resistant staphylococci (C) Gentamicin should be excluded from the regimen (D) Metronidazole should not be administered intravenously (E) The patient should be monitored for candidiasis 4. Which compound is the safest drug to use topically to treat scabies and pediculosis? (A) Benzoyl peroxide (B) Chlorhexidine (C) Lindane (D) Permethrin (E) Silver sulfadiazine 5. Methenamine salts are used as urinary antiseptics. The reason they lack systemic antibacterial action is that they are (A) Converted to formaldehyde only at low pH (B) Metabolized rapidly by hepatic drug-metabolizing enzymes (C) More than 98% bound to plasma proteins (D) Not absorbed into the systemic circulation after oral ingestion (E) Substrates for active tubular secretion 6. Which statement about the actions of antimicrobial agents is false? (A) Metronidazole has activity against C difficile. (B) Neonatal gonococcal ophthalmia can be prevented by silver nitrate (C) Polymyxins act as cationic detergents to disrupt bacterial cell membranes (D) Resistance to nitrofurans emerges rapidly, and there is cross-resistance with sulfonamides (E) Salicylic acid has useful antidermatophytic activity when applied locally 7. Which antiseptic promotes wound healing? (A) Cetylpyridinium chloride (B) Chlorhexidine (C) Hexachlorophene (D) Phenol (E) None of the above 8. A 22-year-old man with gonorrhea is to be treated with cefixime and will need another drug to provide coverage for possible urethritis caused by C trachomatis. Which of the following drugs is least likely to be effective in nongonococcal urethritis? (A) Azithromycin (B) Ciprofloxacin (C) Erythromycin (D) Nitrofurantoin (E) Tetracycline 9. A patient with AIDS has an extremely high viral RNA titer. While blood is being drawn from this patient, the syringe is accidentally dropped, contaminating the floor, which is made of porous material. The best way to deal with this is to (A) Clean the floor with a 10% solution of household bleach (B) Clean the floor with soap and water (C) Completely replace the contaminated part of the floor (D) Neutralize the spill with a solution of potassium permanganate (E) Seal the room and decontaminate with ethylene oxide 10. Neuropathies are more likely to occur with this agent when it is used in patients with renal dysfunction. The drug may cause acute hemolysis in patients with glucose-6-phosphate dehydrogenase (G6PD) deficiency. (A) Chlorhexidine (B) Halazone (C) Methenamine (D) Metronidazole (E) Nitrofurantoin [Show More]

Last updated: 1 year ago

Preview 1 out of 108 pages

Reviews( 0 )

Recommended For You

 Health Care> EXAM REVIEW > Ultrasound Vascular Registry Exam Review with COMPLETE SOLUTION (All)

preview
Ultrasound Vascular Registry Exam Review with COMPLETE SOLUTION

What is the first branch of the aortic arch? - ✔✔Innominate artery/ Brachiocephalic Name the three branches of the aortic arch? - ✔✔a. Innominate artery/ Brachiocephalic b. Lt CCA c. Lt Subclavian...

By Tessa , Uploaded: May 28, 2022

$10

 *NURSING> EXAM REVIEW > ob medsurg mental health review with complete solution (All)

preview
ob medsurg mental health review with complete solution

ob medsurg mental health review with complete solution

By klaus , Uploaded: Apr 05, 2021

$8

 *NURSING> EXAM REVIEW > NURS 612 AHA Exam 2 Study Guide from Chen Walta Review with complete solution (All)

preview
NURS 612 AHA Exam 2 Study Guide from Chen Walta Review with complete solution

HA Exam 2 Study Guide from Chen Walta Review AHA Exam 2 Study Guide from Chen Walta Review Abnormalities in nail beds (pt. complains of problem with nail beds) Fungal infection (onychomycosis) Thick a...

By AplusSuccessor , Uploaded: Apr 01, 2021

$12

 *NURSING> EXAM REVIEW > NR599 Week 8 Final Exam review with complete solution 2021 (All)

preview
NR599 Week 8 Final Exam review with complete solution 2021

NR599 Week 8 Final Exam review with complete solution 2021/NR599 Week 8 Final Exam review with complete solution 2021

By klaus , Uploaded: Feb 28, 2021

$7

 Management> EXAM REVIEW > MGMT 339 Spring 2018 -2020 Final Exam Review with complete solution. (All)

preview
MGMT 339 Spring 2018 -2020 Final Exam Review with complete solution.

MGMT 339 Spring 2018 -2020 Final Exam Review with complete solution. 6K(lean systems),10K(Op planning),11K(resource planning),12K(supply chain design) and 14K(supply chain integration)...

By AplusSuccessor , Uploaded: Dec 12, 2020

$6

 *NURSING> EXAM REVIEW > NURS 612 FINAL EXAM 2020 REVIEW WITH COMPLETE SOLUTION (All)

preview
NURS 612 FINAL EXAM 2020 REVIEW WITH COMPLETE SOLUTION

NURS 612 FINAL EXAM 2020 REVIEW 1.Assessing orientation to person, place, and time helps determine 2.under most conditions, adult patients should be able to repeat a series of ____ numbers. 3.Recen...

By AplusSuccessor , Uploaded: Dec 03, 2020

$7.5

 Business Studies> EXAM REVIEW > AHLEI Exam Review with Complete Solution -checked- Graded A+ (All)

preview
AHLEI Exam Review with Complete Solution -checked- Graded A+

Which of the following statements about hotel food and beverage operations is TRUE? a. Hotel marketing efforts focus exclusively on attracting hotel guests to dine at the property's food and beverage...

By MARKALLAN , Uploaded: Aug 25, 2022

$8

 Health Care> EXAM REVIEW > NHA CCMA EXAM REVIEW With Complete Solution 2023 (All)

preview
NHA CCMA EXAM REVIEW With Complete Solution 2023

NHA CCMA EXAM REVIEW With Complete Solution 2023

By Academic mines , Uploaded: Feb 19, 2023

$10

 Education> EXAM REVIEW > CIW IBA 1.14 OBJECTIVE Exam REVIEW with Complete Solution (All)

preview
CIW IBA 1.14 OBJECTIVE Exam REVIEW with Complete Solution

Which of the following is an example of a trademark? - ANSWER The apple icon on the covers of many Beatles albums Which choice lists the basic elements of authorship, according to copyright law? -...

By Nancylect , Uploaded: Sep 29, 2022

$9

 Medicine> EXAM REVIEW > ENS 304: Exam 1 Review with complete solution Created for 2022 (All)

preview
ENS 304: Exam 1 Review with complete solution Created for 2022

Explain the difference b/n homeostasis and steady state - ANSWER -homeostasis is when stable equilibrium is maintained by physiological processes -steady state is when all variables are constant in s...

By Nancylect , Uploaded: Oct 03, 2022

$7

$7.50

Add to cart

Instant download

Can't find what you want? Try our AI powered Search

OR

GET ASSIGNMENT HELP
175
0

Document information


Connected school, study & course



About the document


Uploaded On

Dec 22, 2020

Number of pages

108

Written in

Seller


seller-icon
AplusSuccessor

Member since 3 years

77 Documents Sold


Additional information

This document has been written for:

Uploaded

Dec 22, 2020

Downloads

 0

Views

 175

Document Keyword Tags

THE BEST STUDY GUIDES

Avoid resits and achieve higher grades with the best study guides, textbook notes, and class notes written by your fellow students

custom preview

Avoid examination resits

Your fellow students know the appropriate material to use to deliver high quality content. With this great service and assistance from fellow students, you can become well prepared and avoid having to resits exams.

custom preview

Get the best grades

Your fellow student knows the best materials to research on and use. This guarantee you the best grades in your examination. Your fellow students use high quality materials, textbooks and notes to ensure high quality

custom preview

Earn from your notes

Get paid by selling your notes and study materials to other students. Earn alot of cash and help other students in study by providing them with appropriate and high quality study materials.

WHAT STUDENTS SAY ABOUT US


What is Browsegrades

In Browsegrades, a student can earn by offering help to other student. Students can help other students with materials by upploading their notes and earn money.

We are here to help

We're available through e-mail, Twitter, Facebook, and live chat.
 FAQ
 Questions? Leave a message!

Follow us on
 Twitter

Copyright © Browsegrades · High quality services·